ABSTRACT

I. SEIZURE CLASSIFICATION (TABLES 12-1 AND 12-2)

A. Seizure 1. Abnormal focal or generalized neuronal discharge, often

with physical manifestations

B. Epilepsy 1. Refers to a condition in which a person experiences two

or more seizures

C. Partial (focal) Seizures 1. Simple partial seizures

a. A spontaneous, uncontrolled neuronal discharge from a focal area of the brain without loss of consciousness

b. Motor seizures: clonic, tonic (asymmetrical, e.g., supplementary motor seizures), automatisms, focal negative myoclonus

c. Sensory seizures 1) Elementary sensory symptoms: a simple sensation

involving one sensory modality, e.g., tingling, a visual sensation, odor, or taste

Table 12-1. Terms Used to Describe Seizures and Their Definitions

Term Definition

Table 12-2. Simplified Outline of the International Classification of Epileptic Seizures

2) Experiential sensory symptoms a) Temporoparieto-occipital junction b) Complex perceptions ± affective experience, similar

to experiences in life but recognized by the subject as occurring out of context, e.g., déjà vu, jamais vu, flashbacks, feelings of depersonalization

d. Autonomic seizures: involve autonomic functions, e.g., unpleasant abdominal sensation, bradycardia, tachycardia, asystole, drooling, piloerection

e. Psychic seizures: disturbance of higher cerebral functions f. Gelastic seizures

1) Bursts of laughter 2) Related to hypothalamic lesions (usually hamartoma)

2. Complex partial seizures a. Spontaneous, uncontrolled neuronal discharge from a

focal area of the brain with loss of consciousness b. Temporal >> frontal > parietal or occipital lobe c. May begin as a simple partial seizure d. May have automatisms: involuntary complex motor

activity during impaired consciousness 1) Examples are gum chewing, nose wiping, drinking

from cup, lip smacking 2) Usually occur with complex seizures, occasionally

with absence seizures 3) Usually occur ipsilateral to the epileptic focus 4) May be de novo automatisms, in which the complex

motor activity begins after the onset of seizure; reactive automatisms, in which the activity also begins after the onset of seizure but is a reaction to an external stimulus; or perseverative automatisms, which are a continuation of the motor activity that was initiated before seizure onset

3. Partial seizures evolving into secondarily generalized seizures

4. Partial status epilepticus a. Epilepsia partialis continua b. Aura continua c. Limbic status epilepticus (psychomotor status) d. Hemiconvulsive status with hemiparesis

D. Generalized Seizures 1. Generalized tonic-clonic seizures

a. With or without premonition (rare, hours-days) b. Tonic phase: eyes open and roll up, pupils dilate, elbows

flex, arms pronate, incontinence, moaning, cyanosis and apnea

c. Clonic phase: generalized clonic movements, frequency gradually decreases, amplitude gradually increases, atonic between jerks, tongue biting, cyanosis and apnea

d. Postictal state: drowsy, confused, lethargy, regular respiration resumes, headache, muscle soreness

2. Generalized tonic seizures a. Axial, proximal limbs, or axial + proximal + distal limbs

involved b. Less common than tonic-clonic seizures c. Typically lasts seconds, can persist (status)

3. Generalized clonic seizures: same clinical significance as generalized tonic-clonic seizures

4. Absence seizures a. Usually occur in children with normal intelligence b. Generalized 3-Hz spike-and-wave electroencephalogram

(EEG) c. Brief duration, usually a few seconds d. Abrupt recovery e. No postictal phase f. Absence with atonic components: primarily affects axial

musculature, causing head drop or slumping of the trunk (falls are rare)

g. Absence with tonic components: asymmetric or symmetric contraction of the flexure or extensor muscles

h. Absence with automatisms: purposeful or semipurposeful movements occurring without awareness (patient is amnestic for the movements)

i. Absence with mild clonic movements: clonic movements may sometimes occur with prolonged spells 1) Are of variable duration and severity 2) Often involve the eyelids or corner of the mouth

j. Absence with impairment of consciousness only: no other features

5. Atypical absence seizures a. Longer duration than typical absence seizures: may last

several minutes b. Less abrupt onset and offset c. Often loss of postural tone (more pronounced than mild

head drop)

Atypical absence seizures differ from absence seizures in that duration is often longer, onset and offset can be less abrupt, and loss of postural tone may be more prominent

Automatisms are involuntary complex motor activity during impaired consciousness

They can occur with complex partial or absence seizures

d. Associated with other seizure types and mental retardation e. Slow generalized spike-and-wave (<2.5 Hz) when seen in

the context of Lennox-Gastaut syndrome (see EPILEPSY SYNDROMES)

f. May have mild clonic, atonic, tonic, or autonomic activity or automatisms

6. Myoclonic seizures a. Shocklike jerk

1) Cortical reflex myoclonus: discharge from sensorimotor cortex

2) Reticular reflex myoclonus: discharge from brainstem reticular formation

3) Primary generalized epileptic myoclonus: diffuse bursts of polyspike and wave or spike and wave

4) Nonepileptic myoclonus: most common 7. Atonic seizures

a. Drop attacks b. Duration: seconds c. Spectrum: from head drop to complete loss of tone in

entire body 8. Akinetic seizures

a. Similar to atonic seizures, but tone is preserved b. Brief loss of consciousness, motionless

9. Infantile spasms (see description under EPILEPSY SYNDROMES)

10. Variations of generalized seizures (examples are myoclonic atonic, massive myoclonic, tonic-clonic beginning with clonic phase)

11. Generalized status epilepticus a. Generalized tonic-clonic seizure b. Clonic seizure c. Absence seizure d. Tonic seizure e. Myoclonic seizure

II. EPILEPSY SYNDROMES (TABLE 12-3)

A. Localization-related (focal, local, partial) Cryptogenic or Symptomatic (secondary) Epilepsy (otherwise unclassified)

1. Temporal lobe seizures

a. Initial behavioral arrest and automatisms (usually with complex partial temporal lobe seizures); initial speech arrest and aphasia (usually with dominant temporal lobe seizures)

b. Associated with epigastric rising sensation, nausea, olfactory hallucinations (usually unpleasant smell, “uncinate fits”), sensation of fear and terror and other changes of affect (intense pleasure or intense depression), gustatory hallucinations (with deep opercular focus), and autonomic symptoms

Epileptic myoclonus includes cortical reflex myoclonus (sensorimotor cortex), reticular reflex myoclonus (reticular formation of brainstem), and primary generalized epileptic myoclonus (diffuse epileptic discharge)

Table 12-3. International Classification of Epilepsy Syndromes

flashes of light, geometric objects (positive or negative visual symptoms)

b. Versive eye movements

B. Neonates 1. Benign neonatal seizures

a. Can be idiopathic with no family history or familial (benign familial neonatal seizures)

b. Also called “fifth-day fits” c. Clinical presentation

1) Clonic or myoclonic seizures and apneic events during first few weeks after birth

2) Seizures usually stop by 6 weeks after birth, no longterm sequelae

3) Normal development 4) Later, some of the children have epilepsy (10%-15%)

or febrile seizures (33%) 5) Treatment is often unnecessary, may prescribe pheno-

barbital for 1 month, then taper d. Generalized epilepsy e. EEG

1) Normal interictally 2) EEG findings are variable

a) There may be focal, multifocal, or bilateral sharp waves, spikes, spike-and-waves

b) There may be a pattern of unreactive, asynchronous theta activity with interspersed spikes (theta point alternant)

f. Benign familial neonatal convulsions is an autosomal dominant channelopathy (Table 12-4)

g. Voltage-dependent potassium channel mutations 1) Mutations in two genes have been identified:

KCNQ2 on chromosome 20 and KCNQ3 on chromosome 8

2) These impair potassium-dependent repolarization, thus causing hyperexcitability

3) Mutation of the KCNQ1 gene causes the long QT syndrome (which also is related to impaired repolarization)

c. May arise from mesial temporal lobe (amygdala, hippocampus, associated with mesial temporal sclerosis) or lateral neocortical temporal lobe

d. Auditory hallucinations (superior temporal gyrus) e. Vertigo and perception of motion f. Memory misperceptions: dreamy state, déjà vu (percep-

tion of familiarity with previously unfamiliar people or events), déjà entendu (perception of unfamiliarity with previously familiar people), jamais vu (perception of familiarity with previously unfamiliar auditory experience), and jamais entendu (perception of unfamiliarity with previously familiar auditory experience)

g. Associated with postictal confusion h. Interictal personality: emotionality, hypermorality and

hyperreligiosity, increased philosophical interest, humorlessness, hypergraphia, circumstantiality of speech, altered libido (hyposexuality more than hypersexuality)

2. Frontal lobe seizures a. Characterized by abrupt onset, brief duration spells

occurring with high frequency (tendency to occur during sleep) with minimal or no postictal confusion

b. Associated with frequent falls during the seizure c. More frequently associated with secondary generalization

and status epilepticus than temporal lobe seizures d. Prominent motor movements such as clonic jerking of

one body part that spreads to involve other body parts, termed “jacksonian march,” because of spread of epileptic activity in motor cortex 1) Head or eyes may turn opposite to side of epileptic

focus 2) Tonic posturing of one limb or “fencer’s posturing”

(see below) is often seen with seizures arising from supplementary motor cortex

3) There may be motor or gestural automatisms, such as bicycling or pedaling movements of the lower limbs, sexual gesturing

e. Most common extratemporal partial epilepsy f. Postictal paralysis (Todd’s paralysis): transient paralysis

that may follow a partial motor seizure 3. Parietal lobe seizures

a. Associated with positive or negative sensory phenomena b. There may be tingling (positive), which often starts in

body parts with larger cortical representation such as the face or the tongue

c. There may be negative phenomena such as asomatognosia (loss of awareness for a body part or whole side of the body) or metamorphopsia (both usually representing a nondominant parietal focus)

4. Occipital lobe seizures a. Elementary visual hallucinations, often bright lines,

Benign familial neonatal convulsions is an autosomal dominant disorder caused by mutation of KCNQ2 or KCNQ3, both genes encoding for potassium channel proteins

These mutations impair potassium-dependent repolarization, resulting in hyperexcitability

2. Early myoclonic encephalopathy a. Also called “early-onset progressive encephalopathy with

migrant, continuous myoclonus” b. Clinical presentation

1) Erratic, focal myoclonus: migrates randomly to different body parts

2) Occurs in early infancy (occasionally within first few hours after birth)

3) The infant may have other seizure types (partial, widespread myoclonus, tonic spasms), but these usually occur later in the course of the disorder

c. Generalized, symptomatic, or cryptogenic 1) Multiple causes, a nonspecific diagnosis

a) Metabolic (nonketotic hyperglycemia) b) Inherited (autosomal recessive) c) Various developmental malformations

d. Often cryptogenic e. EEG

1) Generalized or focal epileptiform discharges 2) EEG often shows burst suppression (as in Ohtahara

syndrome)

3) Myoclonus often does not have an EEG counterpart and EEG may be normal initially

f. Similar to severe myoclonic epilepsy and Ohtahara, Lennox-Gastaut, and West’s syndromes (described below) 1) In all the epileptiform abnormalities thought to con-

tribute to decline in cerebral function 2) Severe psychomotor delay 3) Burst suppression pattern may evolve into hypsar-

rhythmia later in life g. Poor prognosis

1) More than 50% die, others have profound psychomotor delay

3. Ohtahara syndrome a. Also called “early infantile epileptic encephalopathy with

suppression bursts” b. Clinical presentation

1) Frequent tonic spasms ± partial seizures 2) Onset is usually within first 10 days after birth (other-

wise, <3 months) 3) Difficult-to-control seizures 4) Clinical course is marked by neurologic deterioration

c. Symptomatic or cryptogenic: usually structural brain abnormality, multiple causes

d. EEG: burst-suppression pattern e. The EEG is the same in Ohtahara syndrome and early

myoclonic encephalopathy, causing the two to be confused f. Difference: no myoclonic seizures in Ohtahara syndrome g. Seizures are difficult to control, vigabatrin may be benefi-

cial in early stages h. Poor prognosis, 50% die within first few months i. Often progresses to West’s syndrome or Lennox-Gastaut

syndrome phenotypes 4. Migrating partial seizures of infancy

a. Clinical presentation 1) Onset less than 6 months after birth (average, first

seizure at 3 months) 2) Progresses over weeks 3) Multifocal seizures, shift from hemisphere to hemi-

sphere 4) Seizures are nearly continuous at times, occur in clusters 5) Progressive microcephaly and severe psychomotor

deterioration 6) Poor response to anticonvulsants

Table 12-4. Epilepsy Syndromes With Simple Genetic Inheritance

Early myoclonic encephalopathy is characterized by newborn infants with migrant focal myoclonic epilepsy and progressive psychomotor abnormalities

Ohtahara syndrome is early infantile epileptic encephalopathy with tonic spasms and focal seizures associated with a burst-suppression pattern on EEG

b. Idiopathic: no identifiable cause 5. Pyridoxine (vitamin B6)-dependent seizures (congenital

dependency on pyridoxine) a. Autosomal recessive disorder b. Some data suggest this condition results from dimin-

ished activity of glutamic acid decarboxylase (GAD); diminished action of GAD leads to increased cerebral concentrations of glutamic acid, which may not normalize after initiation of treatment with doses necessary to stop seizures

c. Age at onset: usually neonatal period but may appear up to 1 year of age

d. Diagnosis: established by response (remission of seizures) to treatment with parenteral pyridoxine and relapse without ongoing treatment

e. Treatment: life-long administration of pyridoxine oral supplements daily 1) If left untreated, the disease is fatal within days to

months 2) If treatment is delayed, patients develop psychomotor

retardation, progressive deterioration of neurologic function, chronic encephalopathy

f. This condition needs to be differentiated from 1) Pyridoxine deficiency-related seizures: often result

from breastfeeding by malnutritioned mothers and may be recurrent, of abrupt onset, and responsive to vitamin B6 supplementation; in comparison, pyridoxine-dependent seizures occur in setting of normal dietary pyridoxine supplementation, require larger doses of pyridoxine to control seizures, and usually occur earlier than pyridoxine deficiencyrelated seizures

2) Pyridoxine-responsive epilepsy (usually infantile spasms): seizure frequency may decrease with pyridoxine supplementation (in addition to other anticonvulsants)

C. Infants 1. Infantile spasms

a. This is not an epilepsy syndrome but a seizure type

b. May be symptomatic or idiopathic: poor developmental outcome when symptomatic, mild to no mental retardation in 40% when idiopathic

c. Associated with West’s syndrome d. Clinical presentation

1) Occurs within first year after birth 2) Sudden tonic extension or flexion of limbs and axial

body a) Flexion spasms: flexion of neck, trunk, and limbs,

followed by several seconds of tonic activity, or may be mild head droop or waist flexion

b) Extensor spasm: like the Moro reflex 3) Spasms occur in clusters, often after awakening

e. EEG 1) Interictal: hypsarrhythmia (high-amplitude, chaotic

slow waves with multifocal spikes and sharp waves), which diminishes during REM sleep

2) Seizure: electrodecrement (low-amplitude fast activity) f. Treatment: ACTH, vigabatrin (not currently available in

U.S., because of high incidence of retinal toxicity) more effective than other epileptic drugs

g. Benign myoclonus of infancy (described below) can mimic infantile spasms; compared with infantile spasms, benign myoclonus 1) Identical to infantile spasms by semiology but normal

EEG 2) Clusters occur for weeks or months 3) Usually much less frequent by 3 months, none by

2 years 4) Developmentally normal infant 5) Treatment not needed

2. West’s syndrome a. Triad: infantile spasms, hypsarrhythmia, developmental

arrest b. Symptomatic or cryptogenic c. Etiology: several prenatal, perinatal, or postnatal insults,

such as congenital in utero or acquired infections (e.g., meningitis, encephalitis), hydrocephalus, metabolic

Benign myoclonus of infancy appears identical to infantile spasms by semiology, but EEG is normal

Infantile spasms may be idiopathic or symptomatic. When idiopathic, sometimes there is mild or no mental retardation (40%)

Symptomatic infantile spasms are generally associated with poor development

West’s syndrome is a nonspecific diagnosis referring to the triad of infantile spasms, hypsarrhythmia, and developmental arrest

disturbance (postnatal), developmental anomalies (prenatal), tuberous sclerosis, among others (40% cryptogenic)

d. Developmental arrest or regression may occur before seizures develop

e. Treatment: as mentioned, ACTH, corticosteroids, and vigabatrin; the latter is drug of choice for infantile spasms associated with tuberous sclerosis

3. Aicardi’s syndrome a. Triad: Infantile spasms, agenesis of the corpus callosum,

retinal malformations b. X-linked: occurs predominantly in girls, lethal in boys

4. Benign myoclonic epilepsy of infancy a. Clinical presentation

1) Normal infant or toddler (4 months-3 years) 2) Spectrum: from subtle head drop to massive wide-

spread generalized myoclonus (less intense than infantile spasm)

3) Some seizures provoked by intermittent photic stimulation

b. Idiopathic, generalized c. EEG

1) Normal interictally 2) Generalized spike and polyspike with jerks

d. If treated, excellent developmental outcome; some patients have photosensitivity, and some may eventually develop generalized tonic-clonic seizures

e. Easily controlled with valproate f. Similar to a younger version of juvenile myoclonic

epilepsy g. One-third of infants have a family history (as expected

because it is idiopathic, often genetic) 5. Benign infantile seizures

a. Can be subdivided into “benign familial infantile seizures” and “benign nonfamilial infantile seizures”

b. Clinical presentation 1) Partial seizures in first 1 to 2 years after birth 2) Often occur in clusters 1 to 3 days, <10/day 3) Seizures last a maximum of a few minutes 4) No postictal stupor or status 5) Normal psychomotor development

c. Idiopathic 1) Benign epilepsies typically are idiopathic (e.g., benign

myoclonic epilepsy of infancy), but idiopathic epilepsies are not always benign (e.g., severe myoclonic epilepsy of infancy below)

d. Usually autosomal dominant inheritance (when familial) 1) Genetic homogeneity 2) Associated with familial choreoathetosis during

infancy or childhood e. EEG

1) Focal epileptiform discharges 2) May secondarily generalize

f. Treatment: responds well to anticonvulsants 6. Severe myoclonic epilepsy in infancy

a. Also called “Dravet syndrome” b. Clinical presentation

1) Begins within 1 year after birth 2) No previous brain abnormality except occasionally

diffuse atrophy 3) Myoclonic seizures (begin mild, worsen over time) 4) Partial seizures develop later 5) Often, the first seizure occurs with fever, can have

prolonged febrile seizures (i.e., can evolve from febrile seizures)

6) One-fourth of the infants have a family history of seizures

7) Developmental delay with psychomotor regression due to severe, progressive neurologic deterioration that may be secondary to recurrent seizures

c. Idiopathic, generalized or focal d. EEG

1) General, focal, and multifocal abnormalities 2) May be normal interictally (early in disease course) 3) Photosensitivity is common

e. Treatment 1) Seizures are often medically refractory 2) Valproate and benzodiazepines may be tried

D. Children 1. Benign childhood epilepsy with centrotemporal spikes

a. Also called “benign rolandic epilepsy of childhood” b. Common: accounts for one-fourth of childhood seizures c. Clinical presentation

Aicardi’s syndrome is the X-linked triad of infantile spasms, agenesis of corpus callosum, and retinal malformations

Benign epilepsy syndromes are generally idiopathic

But not all idiopathic epilepsies are benign, e.g., idiopathic Dravet syndrome (severe myoclonic epilepsy in infancy) causes progressive brain damage

1) Onset in childhood, age 4 to 12 years 2) Resolves by middle teens 3) Motor, sensory simple seizures: tonic-clonic move-

ments of face or hand, paresthesias of face or hand, drooling, tingling in mouth, speech arrest

4) Can have secondary generalization, usually nocturnal 5) Seizures increase with sleep: 70% of patients have

seizures only during sleep (15% awake only, 15% awake and sleep)

6) Normal development and neurologic examination d. Idiopathic, focal e. EEG

1) Centrotemporal spikes: between central and mid temporal leads (Fig. 12-1)

2) Normal background f. Autosomal dominant inheritance, variable penetrance:

although half of the close relatives of the patient may demonstrate the EEG abnormality during childhood, only 12% of them have clinical seizures

g. Treatment 1) Easily controlled with anticonvulsants

2) Often not necessary to treat (physicians often wait until the second seizure)

3) Treatment can be stopped after adolescence (only 10% of patients continue to have seizures 5 years after onset)

2. Early-onset benign childhood occipital epilepsy

Benign childhood epilepsy with centrotemporal spikes is an example of focal idiopathic epilepsy

Early-onset benign childhood occipital epilepsy involves infrequent seizures (autonomic, hemiconvulsive, and generalized)

Late-onset childhood occipital epilepsy is characterized by frequent visual seizures

a. Also called “Panayiotopoulos syndrome,” “epilepsy associated with ictal vomiting,” “childhood epilepsy with occipital paroxysms”

b. Clinical presentation 1) Most common in children 3 to 6 years old 2) Autonomic seizures and status epilepticus: commonly,

ictal vomiting, eye deviation; often progress to partial clonic or generalized tonic-clonic seizures (often nocturnal)

3) Visual seizures: elementary or complex visual hallucinations, amaurosis, illusions (such as metamorphopsia), which are often experienced during wakefulness

4) Infrequent seizures: most patients, 1 to 3 seizures total 5) Autonomic status epilepticus in almost half of the

seizures c. Overlap with benign childhood epilepsy with centrotem-

poral spikes (also an idiopathic, benign partial epilepsy) 1) Excellent response to anticonvulsants 2) Variable localization of seizures: frequently extra-

occipital, the clinical presentation defines the syndrome rather than occipital spikes

d. EEG 1) Interictal EEG: frequent or nearly continuous bursts

or trains of high-voltage rhythmic occipital spikes and spike-wave complexes at a frequency of 1 to 3 Hz, localized to unilateral or bilateral occipital regions, with normal background activity, increases during non-REM sleep, and disappears with eye opening

2) Ictal EEG: low-voltage fast activity (unilateral or bilateral)

e. Treatment 1) Not needed if only one seizure or a few brief seizures 2) Carbamazepine is usually the first-line treatment

3. Late-onset childhood occipital epilepsy (Gastaut type) a. Clinical presentation

1) Children 4 to 8 years old 2) Visual seizures

a) Hallucinations, blindness: weekly to several per day b) May generalize (rarely) c) Often followed by migraine headache d) Often induced by photic stimulation

3) The children commonly have a family history of benign childhood epilepsy with centrotemporal spikes

4) Both late-onset childhood occipital epilepsy and benign childhood epilepsy with centrotemporal spikes may be benign childhood seizure-susceptibility syndromes with overlapping causes

b. Idiopathic, benign partial epilepsy c. EEG: same as in early-onset benign childhood occipital

epilepsy

d. Treatment is recommended because seizures are frequent 4. Epilepsy with myoclonic absences

a. Rare, with unknown cause b. Clinical presentation

1) Children, average age at onset is 7 years 2) One-half of the children have developmental delay at

time of onset 3) Prolonged (10-60 seconds) absence seizures are

accompanied by bilateral, severe limb myoclonus (may progress to tonic activity) a) Myoclonus is rhythmic, corresponds to the spikes

of the 3-Hz spike-and-wave EEG pattern b) Unlike other absence syndromes, no eye twitching

4) Most children develop other seizure types a) Generalized tonic-clonic seizures b) Typical absence seizures c) Falls

c. Significance 1) Mental impairment is more common than in other

childhood absence syndromes 2) Mental deterioration is thought to be due to seizures 3) One-half of the patients continue to have seizures as

adults d. High-dose ethosuximide and valproate usually control

the seizures e. EEG

1) Ictal: 3-Hz spike-and-wave pattern 2) Interictal: intermittent bursts of generalized spike-

and-waves on a normal background 5. Myoclonic-astatic epilepsy of childhood

a. Clinical presentation 1) The first seizure (often a generalized tonic-clonic

seizure) usually occurs in a developmentally normal child 2 to 5 years old

2) Repeated, sometimes prolonged generalized tonicclonic seizures

3) After months of repeated generalized tonic-clonic seizures, other seizure types (myoclonic, absence, and drop-attacks) appear a) Drop attacks are myoclonic or atonic seizures b) Tonic seizures are uncommon (in contrast to

Epilepsy with myoclonic absences is characterized by long absence seizures with bilateral limb myoclonus

However, unlike atypical absence seizures, EEG shows 3-Hz spike-and-wave activity

Lennox-Gastaut syndrome, in which tonic seizures are a prominent seizure type)

b. Idiopathic, likely polygenic c. EEG: 2 to 3-Hz spike waves (often faster than in

Lennox-Gastaut syndrome) d. Treatment: valproate, ethosuximide, benzodiazepine,

lamotrigine e. Some authorities suspect this is a mild or early form of

Lennox-Gastaut syndrome 6. Lennox-Gastaut syndrome

a. Clinical triad: mental retardation, characteristic slow spike-and-wave (2 Hz) EEG, multiple seizure types

b. Clinical presentation 1) Children, age at onset is 2 to 8 years 2) Boys affected more often than girls 3) The first seizure type is usually drop attacks 4) Many patients have severe mental retardation preced-

ing onset of seizures 5) Later, multiple seizure types evolve, often in associa-

tion with status epilepticus, progressive psychomotor deterioration a) Tonic seizures (last a few seconds)

i) Head/neck flexion or ii) Neck extension/arm abduction or iii) Generalized tonic stiffening sudden fall

b) Atypical absence seizure i) Gradual onset and offset (unlike typical absence) ii) Longer duration than a typical absence seizure iii) May be brief postictal decrease in alertness

(unlike typical absence seizure) c) Atonic seizure: neck only or whole body d) Generalized tonic-clonic seizure e) Less common seizure types: partial tonic-clonic,

myoclonic f) Consider myoclonic-astatic epilepsy if prominent

myoclonic seizures c. Cryptogenic or symptomatic d. Prognosis is usually poor, especially if symptomatic e. Progressive deterioration is thought to be related to fre-

quent subclinical epileptic discharges (epileptic encephalopathy)

f. EEG 1) Interictal “slow spike-and-wave”: double meaning of

name (Table 12-5) a) Spikes are slow (150 ms, longer than a true spike,

which should be <70 ms) b) Spike-and-wave rate is also slow (1.5-2.5 Hz) com-

pared with typical absence seizures (3 Hz) 2) Ictal

a) Tonic seizure: rhythmic fast activity followed by

high-amplitude slow activity b) Absence seizure: slow spike-and-wave discharge

g. Treatment 1) Valproate: all seizure types 2) Lamotrigine, felbamate: especially for drop attacks 3) Carbamazepine and phenytoin: may help generalized

tonic-clonic seizures but may worsen atypical absence seizures

7. Landau-Kleffner syndrome a. Clinical presentation

1) Acquired aphasia (word deafness): the main feature of the syndrome

2) Seizures, may be multiple types: 20% of patients do not have seizures a) Generalized tonic-clonic seizures b) Partial seizures

Table 12-5. Syndromes Associated With Characteristic EEG Patterns

Syndrome EEG pattern

The “slow spike-and-wave” pattern characterizes Lennox-Gastaut syndrome, occurring during atypical absence seizures and interictally

“Slow” refers to both the prolonged duration of “spikes” (150 ms) and the slow rate of spike-andwave activity (1.5-2.5 Hz)

Carbamazepine and phenytoin can worsen atypical absence seizures

Landau-Kleffner syndrome refers to acquired epileptic aphasia in children

c) Myoclonic seizures 3) Children, age at onset: 3 to 8 years old

b. Symptomatic, nonspecific (variety of lesions) c. Magnetic resonance imaging (MRI): usually normal;

functional imaging shows temporal abnormalities d. EEG: variable, multifocal spikes, most commonly temporal e. Outcome is variable

1) Seizures usually are controlled with medication, seizure disorder resolves with time

2) Persistent language problems in about half the children f. Treatment

1) Antiepileptic drugs (valproate and lamotrigine) may help decrease seizure frequency and improve cognitive function

2) Corticosteroids have been tried with some success in small series, but data are inadequate

8. Epilepsy with continuous spike-and-wave pattern during slow wave sleep, also referred to as “electrical status epilepticus during sleep” a. Clinical presentation

1) First seizure occurs in childhood (peak age at onset: 5 years)

2) Multiple seizure types, partial or generalized 3) Seizures occur infrequently, often during sleep 4) Then, seizures accelerate and EEG changes to charac-

teristic pattern (electrical status during slow wave sleep)

5) Psychomotor deterioration (language and motor): thought to be due to seizures

6) Seizures usually resolve by teen years but various degrees of psychomotor abnormalities remain

b. EEG 1) Diffuse or focal interictal discharges (awake) 2) Continuous spike-and-wave pattern during NREM

sleep c. Landau-Kleffner syndrome could be a form of this syn-

drome (affecting language area) because the two syndromes often have a similar sleep EEG

d. Treatment: same as for Landau-Kleffner syndrome 9. Childhood absence epilepsy

a. Clinical presentation 1) Children (girls, 70%), peak age at onset: 6 years 2) Neurologically and developmentally normal 3) Multiple daily spells usually lasting a few seconds

a) Seizures begin and end abruptly b) They completely interrupt activity c) Often, a blank stare d) May have automatisms e) Spells often can be provoked, especially hypo-

glycemia and hyperventilation (which are some-

times used clinically to provoke a spell) 4) Other seizure types

a) About one-third of the children have generalized tonic-clonic seizures later in adolescence (this does not change the diagnosis), but generalized tonicclonic seizures or myoclonic seizures during treatment do not fit the syndrome definition of childhood absence seizures and carry a worse prognosis

b) Mild ictal jerks of lids, eyes, or eyebrows may occur during the first few seconds of a spell, but any more prominent myoclonus such as perioral myoclonus suggests another syndrome, often with a worse prognosis (e.g., epilepsy with myoclonic absence)

b. Idiopathic 1) Unknown genetic cause, but strong genetic predispo-

sition: family may have history of absence or generalized tonic-clonic seizures

2) Atypical absence seizures suggest symptomatic epilepsies such as Lennox-Gastaut syndrome

3) Typical absence seizures can occur in other types of idiopathic and cryptogenic generalized epilepsy syndromes (e.g., juvenile absence seizures)

4) 80% have remission by adulthood c. EEG

1) 3-Hz generalized spike-and-wave pattern: may begin faster (up to 4 Hz) and slow at the end to 2.5 Hz

2) Symmetrical, bilateral, synchronous 3) May be frontal predominant 4) Normal background activity 5) Activated by hyperventilation and hypoglycemia (but

photic stimulation induces spike-and-wave pattern in only 10%-30% of patients)

6) Discharge generator: thalamus 7) Low-threshold (T-type) calcium channels drive the

discharges

Hypoglycemia and hyperventilation can often provoke absence seizures

Generalized tonic-clonic seizures can occur in childhood absence epilepsy, but this occurs later in adolescence

Early tonic-clonic seizures suggest another diagnosis

a) Ethosuximide acts via T-type calcium channel inhibition

b) γ-Aminobutyric acid (GABA)B receptors promote T-type calcium channels

c) Thus, GABAergic drugs (vigabatrin, tiagabine) promote absence seizures

8) Differentiating childhood absence epilepsy from a) Juvenile absence epilepsy

i) Age at onset of childhood absence epilepsy is usually <10 years old and for onset of juvenile absence, 10 to 16 years, but some overlap

ii) More often generalized tonic-clonic seizures in juvenile absence epilepsy, although common also in childhood absence epilepsy

iii) Sometimes myoclonic jerks occur in juvenile absence epilepsy

iv) EEG: 3-Hz spike-and-wave activity, but it may have more polyspikes in juvenile absence epilepsy

v) Juvenile form: somewhat worse prognosis, the child is less likely to outgrow the disease

b) Juvenile myoclonic epilepsy i) Some of the children have absence seizures, but

this is a very different syndrome clinically ii) Myoclonic jerks on awakening iii) No 3-Hz spike-and-wave pattern

d. Prognosis 1) More than 90% of the children outgrow the seizures 2) About one-third have generalized tonic-clonic seizures

later e. Treatment

1) First-line treatment: ethosuximide, valproate, lamotrigine a) Treatment with ethosuximide only prevents

absence seizures, it does not prevent generalized tonic-clonic seizures in children with childhood absence epilepsy

b) Valproate and lamotrigine are effective for both generalized tonic-clonic seizures and absence seizures and are considered the drugs of choice in this situation

2) If monotherapy fails, combination therapy with valproate and ethosuximide

3) Anticonvulsant therapy should be stopped if the EEG is normal and the child has not had seizures for 1 to 2 years

10. Progressive myoclonic epilepsies (Table 12-6) a. Encompasses several progressive disorders, most are

lysosomal and mitochondrial disorders b. Clinical presentation

1) Progressive cognitive deterioration 2) Myoclonus (nonepileptic) 3) Seizures: tonic-clonic, tonic, or myoclonic 4) With or without ataxia, movement disorders

c. Treatment of seizures 1) First-line treatment: valproate 2) Clonazepam and lamotrigine are also used

11. Generalized epilepsy with febrile seizures plus (GEFS+) a. Clinical presentation

1) Febrile seizures 2) “Plus”: this indicates that GEFS+ occurs after age 6

years (unlike typical febrile seizures) or is associated with afebrile generalized tonic-clonic seizures (unlike typical febrile seizures)

3) One-third of patients have other seizure types: absence, myoclonic, atonic, partial seizures

b. Autosomal dominant channelopathy (Table 12-4) 1) Sodium channel (SCN) or GABAA receptor:

increased inward sodium current or decreased GABAmediated inhibition both lead to neuronal hyperexcitability a) SCN1B (chromosome 19) b) SCN1A (chromosome 2) c) SCN2A d) GABAA receptor

Ethosuximide is effective only for absence seizures

Table 12-6. Progressive Myoclonic Epilepsies

Absence seizures are driven by T-type calcium channels of the thalamus, which are blocked by ethosuximide and promoted by GABAergic drugs

Therefore, GABAergic anticonvulsants may promote absence seizures

2) Most febrile seizures, unlike GEFS+, show complex inheritance

c. EEG 1) Generalized spike-and-wave or polyspike-and-wave

pattern 12. Rasmussen’s encephalitis

a. Syndrome of chronic encephalitis with epilepsy in children

b. Intractable, progressive focal seizures; progressive hemiparesis; and cognitive deterioration

c. Radiographic characteristics of slowly progressive cortical atrophy (unilateral more common than bilateral)

d. Antibodies to GLUR3 (glutamate receptor-3) have been implicated in pathogenesis of this disorder

E. Juveniles and Adults 1. Idiopathic generalized epilepsies

a. General category in ILAE classification scheme encompassing 1) Juvenile absence epilepsy 2) Juvenile myoclonic epilepsy 3) Epilepsy with generalized tonic-clonic seizures only

2. Juvenile absence epilepsy a. Clinical presentation

1) Onset at age 10 to 17 years 2) Developmentally normal children 3) Boys and girls affected equally 4) Initially, infrequent absence seizures: usually not daily

(unlike childhood absence epilepsy) 5) Later, tonic-clonic seizures in 75% of patients upon

awakening: more common than in childhood absence epilepsy

b. Idiopathic 1) Genetic mechanism is not known 2) One-third of patients have a family history that may

include childhood absence epilepsy, juvenile absence epilepsy, juvenile myoclonic epilepsy, or epilepsy with grand mal seizures on awakening

c. EEG 1) Same as childhood absence epilepsy, except spike-and-

wave pattern may be slightly faster (3.5-4.5 Hz) and polyspikes are more frequent

d. Prognosis

1) Good: most patients are seizure-free with valproate (controls both absence and generalized seizures), ethosuximide, lamotrigine

2) Patients are less likely to outgrow seizures than those with childhood absence epilepsy a) Patients typically require lifelong anticonvulsant

therapy, and to prevent seizures as adults, patients need to avoid sleep deprivation and alcohol (as with juvenile myoclonic epilepsy)

b) Occurrence of myoclonic seizures and generalized tonic-clonic seizures make seizure remission less likely (overlaps with juvenile myoclonic epilepsy)

3. Juvenile myoclonic epilepsy a. Clinical presentation

1) Age at onset is 8 to 24 years (peak, in teens) 2) Developmentally normal 3) Boys and girls affected equally 4) Myoclonic seizures: the most frequent seizure type

a) Usually predominantly on awakening (early morning or nap)

b) Large-amplitude, bilateral, simultaneous i) Involves both arms or both legs (can cause falls) ii) Unlike nonepileptic myoclonus, it is usually

focal c) May be repetitive d) No loss of awareness e) Often, reflex seizures, including perioral reflex

myoclonia (triggered by reading, talking) 5) Most patients have less frequent generalized tonic-

clonic seizures a) Also, usually occur on awakening b) Often lead to diagnosis in patients with undiag-

nosed history of myoclonus on awakening 6) Some patients have typical absence seizures 7) Seizures are provoked by sleep deprivation, alcohol,

photic stimulation b. Idiopathic

1) One-half of patients have a family history of seizures 2) None are symptomatic (MRI is always normal) 3) No consistent genetic cause has been identified

Juvenile absence epilepsy is characterized by less frequent absence seizures than childhood absence epilepsy and more frequent (75%) development later of tonic-clonic seizures

Like juvenile myoclonic epilepsy, juvenile absence epilepsy usually requires lifelong anticonvulsant therapy and avoidance of triggers (sleep deprivation, alcohol)

Childhood absence epilepsy, in contrast, is usually outgrown

4) Overlap in the same family with other generalized epilepsy syndromes: juvenile absence epilepsy, childhood absence epilepsy, and epilepsy with grand mal seizures on awakening

c. EEG 1) Interictal

a) Three-fourths of patients have generalized 4 to 6-Hz polyspike-and-wave discharges

b) If absence seizures, EEG also has 3-Hz spike-andwave pattern

c) Often photosensitivity 2) Ictal: trains of spikes

d. Treatment 1) First-line: valproate (three-fourths of patients are

seizure-free), it controls all seizure types 2) Second-line: lamotrigine, levetiracetam, topiramate,

and zonisamide 3) Avoid carbamazepine and phenytoin, which are useful

in focal seizures but may worsen some primary generalized seizures (myoclonic and absence seizures)

e. Prognosis: it is a lifelong seizure disorder typically requiring lifelong treatment and avoidance of triggers

4. Epilepsy with grand mal seizures on awakening a. Clinical presentation

1) Onset is in second decade 2) Overlaps clinically with juvenile myoclonic epilepsy

and juvenile absence epilepsy 3) Primary seizure type is generalized tonic-clonic, usually

on awakening 4) Myoclonic or absence seizures may occur but are not

predominant b. Idiopathic/familial c. EEG, treatment, and prognosis: similar to those of

juvenile myoclonic epilepsy 5. Idiopathic photosensitive occipital lobe epilepsy

a. Clinical presentation 1) Reflex epilepsy, begins about the time of puberty 2) Occipital lobe seizures provoked by visual stimuli,

usually television or video games 3) Positive visual symptoms (occipital seizures)

a) The likely origin is the calcarine cortex b) Often, colorful rings or spots

c) May be followed by postictal negative visual symptoms d) Head turns may occur toward the visual phenomena

4) Often spreads to cause autonomic symptoms a) Epigastric discomfort, vomiting b) Can mimic migraine

5) Occasional secondarily generalized seizures b. Idiopathic c. EEG

1) Normal background 2) Interictal epileptiform activity: unilateral or bilateral

occipital ± generalized spike-and-wave, enhanced with eye closure

3) Photoparoxysmal response (epileptiform abnormalities with strobe light) is common

d. Management 1) Avoid triggers 2) First-line treatment: often valproate (as for other pri-

mary generalized epilepsies) 6. Other types of reflex seizures

a. Generalized (tonic-clonic and absence) seizures can be triggered by a flash or visual pattern 1) The most common reflex epilepsy 2) Multiple idiopathic or symptomatic generalized

epilepsy syndromes can have visually evoked seizures, these are not specific to one syndrome

b. Primary reading epilepsy: seizures occur only when patient is reading

c. Startle epilepsy 1) Seizures with unexpected sensory stimuli 2) Usually frequent seizures (tonic or clonic) 3) Often associated with developmental delay

7. Temporal lobe epilepsies a. Traditional view: temporal lobe epilepsy is usually an

acquired, symptomatic epilepsy 1) Most commonly associated with mesial temporal sclerosis 2) One-third of patients have a history of febrile seizures 3) Patients often have a familial predisposition to epilepsy,

but it is usually polygenic and/or multifactorial b. Familial forms (monogenic) of temporal lobe epilepsy

also occur 1) Onset is usually in teens or adults 2) Typically less resistant to treatment 3) A specific gene has been identified: autosomal

Focal occipital seizures of the calcarine cortex usually consist of colorful rings or spots, sometimes with head turns toward the visual phenomena

Valproate is usually effective in juvenile myoclonic epilepsy

Carbamazepine and phenytoin can worsen some types of primary generalized seizures (myoclonic, absence)

dominant partial epilepsy with auditory features (Table 12-4) a) LGI1 mutation (leucine-rich, glioma-inactivated

1 gene) in some families b) Clinical presentation: auditory auras precede com-

plex partial and secondarily generalized seizures 8. Autosomal dominant nocturnal frontal lobe epilepsy

a. Mutations in CHRNA4 gene on chromosome 20q and CHRNB2 gene on chromosome 1p, encoding proteins in M2 transmembrane segment of nicotinic acetylcholine receptors: mutations believed to cause increased acetylcholine sensitivity and channel opening (“gain-offunction” mutations)

b. Most patients have onset of seizures in first two decades (anytime between infancy to adulthood; mean age, 10 years)

c. Clusters of brief, stereotypic, nocturnal seizures, which may consist of hyperkinetic bizarre movements with clonic, tonic, or dystonic features, and sometimes, secondary generalization: clinical features similar to nonfamilial frontal lobe epilepsy

d. Seizures typically occur in non-REM sleep e. Few patients also experience stereotypic seizures during

daytime f. Not associated with cognitive deficits g. Decreased frequency and duration of seizures with time

A. Head Turn (Fig. 12-2) 1. Early nonforced head turn: ipsilateral temporal lobe

a. Voluntary-like, often with dystonic posturing of the contralateral extremity

b. May be due to neglect and/or weakness contralateral to the seizure

2. Forced head turn: contralateral hemisphere (except if it occurs after a generalized seizure) a. Prominent contraction of neck muscles, forcing the chin

to point toward the shoulder b. Tends to occur early in frontal lobe seizures, explosive

onset with other motor seizure activity c. Tends to occur later in temporal lobe seizures, as the

seizure secondarily generalizes (“late forced head turn”) d. If a forced head turn occurs after a secondarily generalized

tonic-clonic seizure, it is usually ipsilateral to seizure onset (“late ipsiversion”), probably because of the spread of seizure activity to the hemisphere contralateral to seizure onset

B. Eye Deviation 1. Accompanies forced head turns (same direction and

upward) 2. Eye deviation in isolation at seizure onset suggests

seizure activity in the occipital lobe, contralateral to the direction of gaze

C. Focal Clonic 1. Frontal and perirolandic area, contralateral hemisphere 2. Can also occur in temporal lobe complex partial

seizures-because of spread of seizure to the surrounding extratemporal cortex

D. Focal Tonic 1. Extended limb 2. Contralateral hemisphere

E. Figure 4 Sign (Fig. 12-3) 1. A tonic limb posture with one arm extended and the

Head turns may be ipsilateral to seizure onset (early nonforced head turn) or contralateral to seizure onset (forced head turn)

Autosomal dominant partial epilepsy with auditory features is an example of a monogenic temporal lobe epilepsy, caused by a mutation of the LGI1 (leucine-rich, glioma-inactivated 1) gene

other flexed at the elbow (forming a “figure 4”) 2. The seizure is lateralized to the hemisphere contralateral

to the extended arm (as with a focal tonic seizure) 3. Figure 4 sign usually occurs as the seizure generalizes 4. A forced head turn may occur (toward the extended arm)

F. Focal Dystonic 1. Sustained contorted posturing of a limb 2. Contralateral hemisphere, possibly due to spread of the

seizure to the basal ganglia

G. Fencing Posture (Fig. 12-4) 1. Seizures localize to hemisphere contralateral to the

extended arm, frontal lobe more than temporal lobe (supplementary motor area)

2. Lateral abduction and external rotation of the arm at the shoulder ± forced head turn to the side of abducted arm

3. Elbow may also flex so the hand is raised to the face

H. Ictal Paresis and Unilateral Immobile Limb 1. Seizures lateralize to contralateral hemisphere

I. Todd’s Paralysis 1. Seizures lateralize to contralateral hemisphere 2. Extratemporal lobe > temporal lobe

J. Unilateral Blinking 1. Seizures lateralize to ipsilateral hemisphere 2. Appears like winks (not forceful like a clonic facial seizure)

K. Unilateral Limb Automatism 1. Seizures lateralize to ipsilateral hemisphere 2. Simple or complex automatic behavior

a. Examples include using a pen, pulling up blanket b. The contralateral side does not take part, possibly

because of neglect

L. Postictal Nose Rubbing 1. Seizures lateralize to hemisphere ipsilateral to hand used 2. Temporal lobe more than frontal lobe

M. Postictal Cough 1. Seizures localize to temporal lobe

N. Bipedal Automatism 1. Frontal lobe more than temporal lobe

Seizures localize to the hemisphere contralateral to the extended arm in both the figure 4 sign and fencing posture

2. Bicycling or kicking

O. Hypermotor 1. Violent, restless thrashing of extremities (“hypermotor

seizures”) suggestive of frontal lobe seizures (supplementary motor area)

2. In contrast, head-and-neck thrashing suggests nonepileptic behavioral event (“pseudoseizure”)

P. Gelastic (laughter) 1. Seizures localize to the hypothalamus or mesial tempo-

ral lobe

Q. Ictal Spitting 1. Seizures localize to the right temporal lobe

R. Ictal Vomiting or Retching 1. Seizures localize to the right temporal lobe

S. Loud Vocalization 1. Frontal lobe more than temporal lobe 2. Often, nonspeech vocalization (grunting, screaming,

moaning) with frontal lobe seizures

T. Speech Arrest 1. Temporal lobe, poorly lateralizing to language-domi-

nant hemisphere 2. Speech preservation during a temporal lobe seizure sug-

gests localization in the nondominant hemisphere, but it can also occur with a dominant hemisphere seizure sparing speech areas-so speech preservation can be misleading

U. Postictal Aphasia 1. Seizures lateralize to the dominant hemisphere

V. Ictal Drooling 1. Seizures lateralize to the nondominant hemisphere

W. Postictal Confusion 1. Is usually more prominent, and persists longer after

temporal lobe seizures 2. Is not present or is brief and less prominent after

frontal lobe seizures

X. Visual Symptoms 1. Elementary: occipital lobe

a. Positive visual symptoms > negative ones but either is possible 1) Circles, spots, shapes, flashes, hemianopsia, scotoma

2) Often in color b. Pattern and progression may be stereotyped for an indi-

vidual patient c. Brief (seconds), with movement and changing shape and

size d. There may be a postictal headache, can be misdiagnosed

as migraine 1) Migraine aura has more colorful, rounded forms or

shapes rather than linear or zigzag lines 2) Migraine aura is usually briefer (seconds)

2. Complex visual hallucinations or illusions: occipitotemporoparietal junction of nondominant parietal lobe a. Relatively rare b. Are usually complex and colorful, are often previously

experienced images c. Can include micropsia (images appear smaller), meta-

morphopsia (objects appear distorted)

A. Phenytoin 1. Seizure types

a. Partial or generalized tonic-clonic seizures (primary or secondary)

b. Rarely, phenytoin can worsen other types of generalized seizures (myoclonic, absence)

2. Mechanism of action: inhibits sodium channels, especially at high rates of firing

3. Metabolism

Postictal confusion is most prominent after temporal lobe seizures and may be absent after frontal lobe seizures

Elementary visual symptoms suggest occipital lobe localization, whereas complex visual hallucinations or illusions localize to the nondominant parietal lobe or occipitotemporoparietal junction

Unilateral limb automatisms, including nose rubbing, correspond to a seizure focus ipsilateral to the limb used

a. Liver (minimal renal excretion) b. Monitor for toxicity, especially if liver disease c. May need to follow free plasma concentrations in renal

disease because of less protein binding (total may underestimate free level)

d. Nonlinear kinetics: saturable, and concentrationdependent, nonlinear (zero-order) elimination kinetics 1) As plasma concentration of the drug increases, the

elimination mechanism is progressively saturated in a nonlinear fashion, resulting in a disproportionate and logarithmic increase in plasma drug concentrations

2) Thus, small additional doses may cause a large increment in plasma concentrations when the elimination mechanism is saturated

4. Side effects a. Idiosyncratic: dyscrasias, morbilliform rash, Stevens-

Johnson syndrome, hepatitis b. Cosmetic

1) Gingival hyperplasia: lessened by good oral hygiene 2) Probably causes acne, coarse features, and hirsutism

c. Toxic to tissues (because it is highly alkaline) 1) Muscle breakdown if phenytoin is injected intramus-

cularly (therefore, only fosphenytoin is given intramuscularly)

2) Purple glove syndrome a) Severe tissue injury from constriction of blood ves-

sels, can lead to amputation or sepsis b) This occurs with intravenous administration,

occurs much less frequently with fosphenytoin d. Neurotoxicity: because of narrow therapeutic window,

blood levels are useful for monitoring for neurotoxicity (unlike newer anticonvulsants) (“narrow therapeutic window” refers to nonlinear, saturable, and concentration-dependent elimination kinetics of phenytoin, discussed above) 1) Dose-dependent, typically minimal at therapeutic levels 2) Nystagmus (at higher blood concentrations) ataxia,

dysarthria, diplopia, nausea, dizziness drowsiness, cognitive difficulties coma

3) At high levels, may rarely increase seizure frequency 4) Movement disorders

e. Vitamin-related 1) Folate deficiency/anemia 2) Increased vitamin D metabolism: causes premature

osteoporosis f. Chronic

1) Cerebellar atrophy 2) Mild peripheral neuropathy

g. Acute (with intravenous formulation) 1) Phlebitis, pain, burning sensation (with too rapid

infusion) a) This occurs because undiluted administration is

required (injectable phenytoin is insoluble in standard intravenous fluids)

b) In comparison, fosphenytoin is freely soluble in all standard intravenous fluids and thus asssociated with fewer local adverse effects such as irritation and superficial phlebitis and systemic adverse effects such as hypotension

2) Hypotension, conduction abnormalities (need to monitor blood pressure during infusion), especially if cardiac disease

3) Hypotension and conduction abnormalities are less severe with fosphenytoin

5. Drug interactions a. Variable effect in plasma concentration of warfarin b. Decreases plasma concentration of

1) Most anticonvulsants (except for gabapentin and levetiracetam): carbamazepine, oxcarbazepine, topiramate, tiagabine, lamotrigine, zonisamide, valproate, felbamate

2) Other: benzodiazepines, haloperidol, tricyclic antidepressants, oral contraceptives, digoxin, cyclosporine

c. The following increase the plasma concentration of phenytoin: cimetidine, carbamazepine, felbamate, fluconazole, fluoxetine, valproate

d. The following lower the concentration of phenytoin: valproate (decreases total, but increases free levels) and rifampin

6. Monitor: liver function tests 7. Calculating the loading dose if levels are subtherapeutic

(estimate only) a. If phenytoin level is <10 μg/mL, first-order kinetics b. Approaches zero-order kinetics above 10 μg/mL, so small

dose increments produce large increases in blood levels c. For phenytoin level <10 μg/mL, additional dose to achieve

a desired concentration can be estimated as follows: 1) Additional oral dose (mg/kg) = [volume of distribution

(0.7) desired increment (μg/mL) (desired concentration – measured concentration)]/0.92

2) Additional intravenous dose (mg/kg) = 0.7 (desired concentration – measured concentration)

3) Example: if the total phenytoin concentration is 5 μg/mL and the concentration desired is 10 μg/mL,

Phenytoin and carbamazepine act by inhibiting sodium channels, especially at high rates of firing

the additional intravenous dose will be 0.7 (10 μg/mL – 5 μg/mL) = 3.5 mg/kg

B. Fosphenytoin 1. Intravenous prodrug of phenytoin

a. Useful for intravenous load in status epilepticus or if patient is unable to take medication orally

b. Converted to phenytoin by erythrocytes (using alkaline phosphatase)

c. Fosphenytoin is a phosphate ester that is water soluble and less alkaline than phenytoin

d. In contrast, phenytoin is administered in a highly alkaline mixture of propylene glycol (antifreeze) and ethanol and can precipitate if administered too rapidly 1) Thus, fosphenytoin is less toxic to tissue, and no pur-

ple glove syndrome develops 2) Fosphenytoin can be administered more rapidly than

phenytoin and with less hypotension a) Maximal fosphenytoin dose: 150 mg of phenytoin

equivalents per minute b) Maximal phenytoin dose: 50 mg per minute

3) Because it takes 8 to 15 minutes for fosphenytoin to convert to phenytoin, the more rapid administration of fosphenytoin is offset by the time for conversion

4) Both drugs take the same time to achieve equivalent phenytoin concentration

e. Fosphenytoin also can be administered intramuscularly for a loading dose in patient who cannot take medication orally

f. Otherwise, the side effects are identical to those of phenytoin

C. Phenobarbital 1. Seizure types

a. Partial or generalized tonic-clonic seizures (primary or secondary)

b. Phenobarbital is less effective than phenytoin or carbamazepine for partial seizures

2. Mechanisms of action a. GABAA agonist b. Sodium channel antagonist c. T-type calcium channel antagonist d. Glutamate antagonist

3. Metabolism a. Liver with renal excretion of metabolites b. Increased risk of toxicity if patient has kidney or liver

disease 4. Side effects

a. Cognitive (dose-dependent) 1) Sedation, irritability, cognitive difficulties, ataxia 2) Hyperactivity (children) 3) Narrow therapeutic window, so plasma levels are use-

ful in monitoring for toxicity (unlike newer anticonvulsants)

b. Idiosyncratic: rash, aplastic anemia, agranulocytosis, thrombocytopenia, hepatitis

c. Vitamin deficiencies 1) Vitamin D deficiency and bone marrow density 2) Folate, vitamin K deficiencies

d. Cardiac and respiratory depression 5. Drug interactions

a. Decreases the plasma concentration of 1) Most anticonvulsants (except for gabapentin and leve-

tiracetam): valproate, carbamazepine, oxcarbazepine, lamotrigine, topiramate, tiagabine, zonisamide

2) Other: benzodiazepines, haloperidol, theophylline, cimetidine, warfarin, oral contraceptives

b. The following increase the plasma concentration of phenobarbital: tricyclic antidepressants and valproate

c. Cardiac and respiratory depression 6. Monitor: liver function tests

Liver enzyme-inducing anticonvulsants, like phenytoin, can reduce the effectiveness of low-dose oral contraceptives

Valproate, a liver enzyme inhibitor, also can reduce the effectiveness of low-dose oral contraceptives

Fosphenytoin is less toxic to tissue than phenytoin because fosphenytoin is water soluble

Phenytoin, in contrast, is administered in a highly alkaline mixture of propylene glycol and ethanol and can precipitate if administered too rapidly

Liver enzyme-inducing anticonvulsants include phenytoin, phenobarbital, and carbamazepine

Oxcarbazepine has less liver enzyme-induction than carbamazepine

2. Early myoclonic encephalopathy a. Also called “early-onset progressive encephalopathy with

migrant, continuous myoclonus” b. Clinical presentation

1) Erratic, focal myoclonus: migrates randomly to different body parts

2) Occurs in early infancy (occasionally within first few hours after birth)

3) The infant may have other seizure types (partial, widespread myoclonus, tonic spasms), but these usually occur later in the course of the disorder

c. Generalized, symptomatic, or cryptogenic 1) Multiple causes, a nonspecific diagnosis

a) Metabolic (nonketotic hyperglycemia) b) Inherited (autosomal recessive) c) Various developmental malformations

d. Often cryptogenic e. EEG

1) Generalized or focal epileptiform discharges 2) EEG often shows burst suppression (as in Ohtahara

syndrome)

3) Myoclonus often does not have an EEG counterpart and EEG may be normal initially

f. Similar to severe myoclonic epilepsy and Ohtahara, Lennox-Gastaut, and West’s syndromes (described below) 1) In all the epileptiform abnormalities thought to con-

tribute to decline in cerebral function 2) Severe psychomotor delay 3) Burst suppression pattern may evolve into hypsar-

rhythmia later in life g. Poor prognosis

1) More than 50% die, others have profound psychomotor delay

3. Ohtahara syndrome a. Also called “early infantile epileptic encephalopathy with

suppression bursts” b. Clinical presentation

1) Frequent tonic spasms ± partial seizures 2) Onset is usually within first 10 days after birth (other-

wise, <3 months) 3) Difficult-to-control seizures 4) Clinical course is marked by neurologic deterioration

c. Symptomatic or cryptogenic: usually structural brain abnormality, multiple causes

d. EEG: burst-suppression pattern e. The EEG is the same in Ohtahara syndrome and early

myoclonic encephalopathy, causing the two to be confused f. Difference: no myoclonic seizures in Ohtahara syndrome g. Seizures are difficult to control, vigabatrin may be benefi-

cial in early stages h. Poor prognosis, 50% die within first few months i. Often progresses to West’s syndrome or Lennox-Gastaut

syndrome phenotypes 4. Migrating partial seizures of infancy

a. Clinical presentation 1) Onset less than 6 months after birth (average, first

seizure at 3 months) 2) Progresses over weeks 3) Multifocal seizures, shift from hemisphere to hemi-

sphere 4) Seizures are nearly continuous at times, occur in clusters 5) Progressive microcephaly and severe psychomotor

deterioration 6) Poor response to anticonvulsants

Table 12-4. Epilepsy Syndromes With Simple Genetic Inheritance

Early myoclonic encephalopathy is characterized by newborn infants with migrant focal myoclonic epilepsy and progressive psychomotor abnormalities

Ohtahara syndrome is early infantile epileptic encephalopathy with tonic spasms and focal seizures associated with a burst-suppression pattern on EEG

b. Idiopathic: no identifiable cause 5. Pyridoxine (vitamin B6)-dependent seizures (congenital

dependency on pyridoxine) a. Autosomal recessive disorder b. Some data suggest this condition results from dimin-

ished activity of glutamic acid decarboxylase (GAD); diminished action of GAD leads to increased cerebral concentrations of glutamic acid, which may not normalize after initiation of treatment with doses necessary to stop seizures

c. Age at onset: usually neonatal period but may appear up to 1 year of age

d. Diagnosis: established by response (remission of seizures) to treatment with parenteral pyridoxine and relapse without ongoing treatment

e. Treatment: life-long administration of pyridoxine oral supplements daily 1) If left untreated, the disease is fatal within days to

months 2) If treatment is delayed, patients develop psychomotor

retardation, progressive deterioration of neurologic function, chronic encephalopathy

f. This condition needs to be differentiated from 1) Pyridoxine deficiency-related seizures: often result

from breastfeeding by malnutritioned mothers and may be recurrent, of abrupt onset, and responsive to vitamin B6 supplementation; in comparison, pyridoxine-dependent seizures occur in setting of normal dietary pyridoxine supplementation, require larger doses of pyridoxine to control seizures, and usually occur earlier than pyridoxine deficiencyrelated seizures

2) Pyridoxine-responsive epilepsy (usually infantile spasms): seizure frequency may decrease with pyridoxine supplementation (in addition to other anticonvulsants)

C. Infants 1. Infantile spasms

a. This is not an epilepsy syndrome but a seizure type

b. May be symptomatic or idiopathic: poor developmental outcome when symptomatic, mild to no mental retardation in 40% when idiopathic

c. Associated with West’s syndrome d. Clinical presentation

1) Occurs within first year after birth 2) Sudden tonic extension or flexion of limbs and axial

body a) Flexion spasms: flexion of neck, trunk, and limbs,

followed by several seconds of tonic activity, or may be mild head droop or waist flexion

b) Extensor spasm: like the Moro reflex 3) Spasms occur in clusters, often after awakening

e. EEG 1) Interictal: hypsarrhythmia (high-amplitude, chaotic

slow waves with multifocal spikes and sharp waves), which diminishes during REM sleep

2) Seizure: electrodecrement (low-amplitude fast activity) f. Treatment: ACTH, vigabatrin (not currently available in

U.S., because of high incidence of retinal toxicity) more effective than other epileptic drugs

g. Benign myoclonus of infancy (described below) can mimic infantile spasms; compared with infantile spasms, benign myoclonus 1) Identical to infantile spasms by semiology but normal

EEG 2) Clusters occur for weeks or months 3) Usually much less frequent by 3 months, none by

2 years 4) Developmentally normal infant 5) Treatment not needed

2. West’s syndrome a. Triad: infantile spasms, hypsarrhythmia, developmental

arrest b. Symptomatic or cryptogenic c. Etiology: several prenatal, perinatal, or postnatal insults,

such as congenital in utero or acquired infections (e.g., meningitis, encephalitis), hydrocephalus, metabolic

Benign myoclonus of infancy appears identical to infantile spasms by semiology, but EEG is normal

Infantile spasms may be idiopathic or symptomatic. When idiopathic, sometimes there is mild or no mental retardation (40%)

Symptomatic infantile spasms are generally associated with poor development

West’s syndrome is a nonspecific diagnosis referring to the triad of infantile spasms, hypsarrhythmia, and developmental arrest

disturbance (postnatal), developmental anomalies (prenatal), tuberous sclerosis, among others (40% cryptogenic)

d. Developmental arrest or regression may occur before seizures develop

e. Treatment: as mentioned, ACTH, corticosteroids, and vigabatrin; the latter is drug of choice for infantile spasms associated with tuberous sclerosis

3. Aicardi’s syndrome a. Triad: Infantile spasms, agenesis of the corpus callosum,

retinal malformations b. X-linked: occurs predominantly in girls, lethal in boys

4. Benign myoclonic epilepsy of infancy a. Clinical presentation

1) Normal infant or toddler (4 months-3 years) 2) Spectrum: from subtle head drop to massive wide-

spread generalized myoclonus (less intense than infantile spasm)

3) Some seizures provoked by intermittent photic stimulation

b. Idiopathic, generalized c. EEG

1) Normal interictally 2) Generalized spike and polyspike with jerks

d. If treated, excellent developmental outcome; some patients have photosensitivity, and some may eventually develop generalized tonic-clonic seizures

e. Easily controlled with valproate f. Similar to a younger version of juvenile myoclonic

epilepsy g. One-third of infants have a family history (as expected

because it is idiopathic, often genetic) 5. Benign infantile seizures

a. Can be subdivided into “benign familial infantile seizures” and “benign nonfamilial infantile seizures”

b. Clinical presentation 1) Partial seizures in first 1 to 2 years after birth 2) Often occur in clusters 1 to 3 days, <10/day 3) Seizures last a maximum of a few minutes 4) No postictal stupor or status 5) Normal psychomotor development

c. Idiopathic 1) Benign epilepsies typically are idiopathic (e.g., benign

myoclonic epilepsy of infancy), but idiopathic epilepsies are not always benign (e.g., severe myoclonic epilepsy of infancy below)

d. Usually autosomal dominant inheritance (when familial) 1) Genetic homogeneity 2) Associated with familial choreoathetosis during

infancy or childhood e. EEG

1) Focal epileptiform discharges 2) May secondarily generalize

f. Treatment: responds well to anticonvulsants 6. Severe myoclonic epilepsy in infancy

a. Also called “Dravet syndrome” b. Clinical presentation

1) Begins within 1 year after birth 2) No previous brain abnormality except occasionally

diffuse atrophy 3) Myoclonic seizures (begin mild, worsen over time) 4) Partial seizures develop later 5) Often, the first seizure occurs with fever, can have

prolonged febrile seizures (i.e., can evolve from febrile seizures)

6) One-fourth of the infants have a family history of seizures

7) Developmental delay with psychomotor regression due to severe, progressive neurologic deterioration that may be secondary to recurrent seizures

c. Idiopathic, generalized or focal d. EEG

1) General, focal, and multifocal abnormalities 2) May be normal interictally (early in disease course) 3) Photosensitivity is common

e. Treatment 1) Seizures are often medically refractory 2) Valproate and benzodiazepines may be tried

D. Children 1. Benign childhood epilepsy with centrotemporal spikes

a. Also called “benign rolandic epilepsy of childhood” b. Common: accounts for one-fourth of childhood seizures c. Clinical presentation

Aicardi’s syndrome is the X-linked triad of infantile spasms, agenesis of corpus callosum, and retinal malformations

Benign epilepsy syndromes are generally idiopathic

But not all idiopathic epilepsies are benign, e.g., idiopathic Dravet syndrome (severe myoclonic epilepsy in infancy) causes progressive brain damage

1) Onset in childhood, age 4 to 12 years 2) Resolves by middle teens 3) Motor, sensory simple seizures: tonic-clonic move-

ments of face or hand, paresthesias of face or hand, drooling, tingling in mouth, speech arrest

4) Can have secondary generalization, usually nocturnal 5) Seizures increase with sleep: 70% of patients have

seizures only during sleep (15% awake only, 15% awake and sleep)

6) Normal development and neurologic examination d. Idiopathic, focal e. EEG

1) Centrotemporal spikes: between central and mid temporal leads (Fig. 12-1)

2) Normal background f. Autosomal dominant inheritance, variable penetrance:

although half of the close relatives of the patient may demonstrate the EEG abnormality during childhood, only 12% of them have clinical seizures

g. Treatment 1) Easily controlled with anticonvulsants

2) Often not necessary to treat (physicians often wait until the second seizure)

3) Treatment can be stopped after adolescence (only 10% of patients continue to have seizures 5 years after onset)

2. Early-onset benign childhood occipital epilepsy

Benign childhood epilepsy with centrotemporal spikes is an example of focal idiopathic epilepsy

Early-onset benign childhood occipital epilepsy involves infrequent seizures (autonomic, hemiconvulsive, and generalized)

Late-onset childhood occipital epilepsy is characterized by frequent visual seizures

a. Also called “Panayiotopoulos syndrome,” “epilepsy associated with ictal vomiting,” “childhood epilepsy with occipital paroxysms”

b. Clinical presentation 1) Most common in children 3 to 6 years old 2) Autonomic seizures and status epilepticus: commonly,

ictal vomiting, eye deviation; often progress to partial clonic or generalized tonic-clonic seizures (often nocturnal)

3) Visual seizures: elementary or complex visual hallucinations, amaurosis, illusions (such as metamorphopsia), which are often experienced during wakefulness

4) Infrequent seizures: most patients, 1 to 3 seizures total 5) Autonomic status epilepticus in almost half of the

seizures c. Overlap with benign childhood epilepsy with centrotem-

poral spikes (also an idiopathic, benign partial epilepsy) 1) Excellent response to anticonvulsants 2) Variable localization of seizures: frequently extra-

occipital, the clinical presentation defines the syndrome rather than occipital spikes

d. EEG 1) Interictal EEG: frequent or nearly continuous bursts

or trains of high-voltage rhythmic occipital spikes and spike-wave complexes at a frequency of 1 to 3 Hz, localized to unilateral or bilateral occipital regions, with normal background activity, increases during non-REM sleep, and disappears with eye opening

2) Ictal EEG: low-voltage fast activity (unilateral or bilateral)

e. Treatment 1) Not needed if only one seizure or a few brief seizures 2) Carbamazepine is usually the first-line treatment

3. Late-onset childhood occipital epilepsy (Gastaut type) a. Clinical presentation

1) Children 4 to 8 years old 2) Visual seizures

a) Hallucinations, blindness: weekly to several per day b) May generalize (rarely) c) Often followed by migraine headache d) Often induced by photic stimulation

3) The children commonly have a family history of benign childhood epilepsy with centrotemporal spikes

4) Both late-onset childhood occipital epilepsy and benign childhood epilepsy with centrotemporal spikes may be benign childhood seizure-susceptibility syndromes with overlapping causes

b. Idiopathic, benign partial epilepsy c. EEG: same as in early-onset benign childhood occipital

epilepsy

d. Treatment is recommended because seizures are frequent 4. Epilepsy with myoclonic absences

a. Rare, with unknown cause b. Clinical presentation

1) Children, average age at onset is 7 years 2) One-half of the children have developmental delay at

time of onset 3) Prolonged (10-60 seconds) absence seizures are

accompanied by bilateral, severe limb myoclonus (may progress to tonic activity) a) Myoclonus is rhythmic, corresponds to the spikes

of the 3-Hz spike-and-wave EEG pattern b) Unlike other absence syndromes, no eye twitching

4) Most children develop other seizure types a) Generalized tonic-clonic seizures b) Typical absence seizures c) Falls

c. Significance 1) Mental impairment is more common than in other

childhood absence syndromes 2) Mental deterioration is thought to be due to seizures 3) One-half of the patients continue to have seizures as

adults d. High-dose ethosuximide and valproate usually control

the seizures e. EEG

1) Ictal: 3-Hz spike-and-wave pattern 2) Interictal: intermittent bursts of generalized spike-

and-waves on a normal background 5. Myoclonic-astatic epilepsy of childhood

a. Clinical presentation 1) The first seizure (often a generalized tonic-clonic

seizure) usually occurs in a developmentally normal child 2 to 5 years old

2) Repeated, sometimes prolonged generalized tonicclonic seizures

3) After months of repeated generalized tonic-clonic seizures, other seizure types (myoclonic, absence, and drop-attacks) appear a) Drop attacks are myoclonic or atonic seizures b) Tonic seizures are uncommon (in contrast to

Epilepsy with myoclonic absences is characterized by long absence seizures with bilateral limb myoclonus

However, unlike atypical absence seizures, EEG shows 3-Hz spike-and-wave activity

Lennox-Gastaut syndrome, in which tonic seizures are a prominent seizure type)

b. Idiopathic, likely polygenic c. EEG: 2 to 3-Hz spike waves (often faster than in

Lennox-Gastaut syndrome) d. Treatment: valproate, ethosuximide, benzodiazepine,

lamotrigine e. Some authorities suspect this is a mild or early form of

Lennox-Gastaut syndrome 6. Lennox-Gastaut syndrome

a. Clinical triad: mental retardation, characteristic slow spike-and-wave (2 Hz) EEG, multiple seizure types

b. Clinical presentation 1) Children, age at onset is 2 to 8 years 2) Boys affected more often than girls 3) The first seizure type is usually drop attacks 4) Many patients have severe mental retardation preced-

ing onset of seizures 5) Later, multiple seizure types evolve, often in associa-

tion with status epilepticus, progressive psychomotor deterioration a) Tonic seizures (last a few seconds)

i) Head/neck flexion or ii) Neck extension/arm abduction or iii) Generalized tonic stiffening sudden fall

b) Atypical absence seizure i) Gradual onset and offset (unlike typical absence) ii) Longer duration than a typical absence seizure iii) May be brief postictal decrease in alertness

(unlike typical absence seizure) c) Atonic seizure: neck only or whole body d) Generalized tonic-clonic seizure e) Less common seizure types: partial tonic-clonic,

myoclonic f) Consider myoclonic-astatic epilepsy if prominent

myoclonic seizures c. Cryptogenic or symptomatic d. Prognosis is usually poor, especially if symptomatic e. Progressive deterioration is thought to be related to fre-

quent subclinical epileptic discharges (epileptic encephalopathy)

f. EEG 1) Interictal “slow spike-and-wave”: double meaning of

name (Table 12-5) a) Spikes are slow (150 ms, longer than a true spike,

which should be <70 ms) b) Spike-and-wave rate is also slow (1.5-2.5 Hz) com-

pared with typical absence seizures (3 Hz) 2) Ictal

a) Tonic seizure: rhythmic fast activity followed by

high-amplitude slow activity b) Absence seizure: slow spike-and-wave discharge

g. Treatment 1) Valproate: all seizure types 2) Lamotrigine, felbamate: especially for drop attacks 3) Carbamazepine and phenytoin: may help generalized

tonic-clonic seizures but may worsen atypical absence seizures

7. Landau-Kleffner syndrome a. Clinical presentation

1) Acquired aphasia (word deafness): the main feature of the syndrome

2) Seizures, may be multiple types: 20% of patients do not have seizures a) Generalized tonic-clonic seizures b) Partial seizures

Table 12-5. Syndromes Associated With Characteristic EEG Patterns

Syndrome EEG pattern

The “slow spike-and-wave” pattern characterizes Lennox-Gastaut syndrome, occurring during atypical absence seizures and interictally

“Slow” refers to both the prolonged duration of “spikes” (150 ms) and the slow rate of spike-andwave activity (1.5-2.5 Hz)

Carbamazepine and phenytoin can worsen atypical absence seizures

Landau-Kleffner syndrome refers to acquired epileptic aphasia in children

c) Myoclonic seizures 3) Children, age at onset: 3 to 8 years old

b. Symptomatic, nonspecific (variety of lesions) c. Magnetic resonance imaging (MRI): usually normal;

functional imaging shows temporal abnormalities d. EEG: variable, multifocal spikes, most commonly temporal e. Outcome is variable

1) Seizures usually are controlled with medication, seizure disorder resolves with time

2) Persistent language problems in about half the children f. Treatment

1) Antiepileptic drugs (valproate and lamotrigine) may help decrease seizure frequency and improve cognitive function

2) Corticosteroids have been tried with some success in small series, but data are inadequate

8. Epilepsy with continuous spike-and-wave pattern during slow wave sleep, also referred to as “electrical status epilepticus during sleep” a. Clinical presentation

1) First seizure occurs in childhood (peak age at onset: 5 years)

2) Multiple seizure types, partial or generalized 3) Seizures occur infrequently, often during sleep 4) Then, seizures accelerate and EEG changes to charac-

teristic pattern (electrical status during slow wave sleep)

5) Psychomotor deterioration (language and motor): thought to be due to seizures

6) Seizures usually resolve by teen years but various degrees of psychomotor abnormalities remain

b. EEG 1) Diffuse or focal interictal discharges (awake) 2) Continuous spike-and-wave pattern during NREM

sleep c. Landau-Kleffner syndrome could be a form of this syn-

drome (affecting language area) because the two syndromes often have a similar sleep EEG

d. Treatment: same as for Landau-Kleffner syndrome 9. Childhood absence epilepsy

a. Clinical presentation 1) Children (girls, 70%), peak age at onset: 6 years 2) Neurologically and developmentally normal 3) Multiple daily spells usually lasting a few seconds

a) Seizures begin and end abruptly b) They completely interrupt activity c) Often, a blank stare d) May have automatisms e) Spells often can be provoked, especially hypo-

glycemia and hyperventilation (which are some-

times used clinically to provoke a spell) 4) Other seizure types

a) About one-third of the children have generalized tonic-clonic seizures later in adolescence (this does not change the diagnosis), but generalized tonicclonic seizures or myoclonic seizures during treatment do not fit the syndrome definition of childhood absence seizures and carry a worse prognosis

b) Mild ictal jerks of lids, eyes, or eyebrows may occur during the first few seconds of a spell, but any more prominent myoclonus such as perioral myoclonus suggests another syndrome, often with a worse prognosis (e.g., epilepsy with myoclonic absence)

b. Idiopathic 1) Unknown genetic cause, but strong genetic predispo-

sition: family may have history of absence or generalized tonic-clonic seizures

2) Atypical absence seizures suggest symptomatic epilepsies such as Lennox-Gastaut syndrome

3) Typical absence seizures can occur in other types of idiopathic and cryptogenic generalized epilepsy syndromes (e.g., juvenile absence seizures)

4) 80% have remission by adulthood c. EEG

1) 3-Hz generalized spike-and-wave pattern: may begin faster (up to 4 Hz) and slow at the end to 2.5 Hz

2) Symmetrical, bilateral, synchronous 3) May be frontal predominant 4) Normal background activity 5) Activated by hyperventilation and hypoglycemia (but

photic stimulation induces spike-and-wave pattern in only 10%-30% of patients)

6) Discharge generator: thalamus 7) Low-threshold (T-type) calcium channels drive the

discharges

Hypoglycemia and hyperventilation can often provoke absence seizures

Generalized tonic-clonic seizures can occur in childhood absence epilepsy, but this occurs later in adolescence

Early tonic-clonic seizures suggest another diagnosis

a) Ethosuximide acts via T-type calcium channel inhibition

b) γ-Aminobutyric acid (GABA)B receptors promote T-type calcium channels

c) Thus, GABAergic drugs (vigabatrin, tiagabine) promote absence seizures

8) Differentiating childhood absence epilepsy from a) Juvenile absence epilepsy

i) Age at onset of childhood absence epilepsy is usually <10 years old and for onset of juvenile absence, 10 to 16 years, but some overlap

ii) More often generalized tonic-clonic seizures in juvenile absence epilepsy, although common also in childhood absence epilepsy

iii) Sometimes myoclonic jerks occur in juvenile absence epilepsy

iv) EEG: 3-Hz spike-and-wave activity, but it may have more polyspikes in juvenile absence epilepsy

v) Juvenile form: somewhat worse prognosis, the child is less likely to outgrow the disease

b) Juvenile myoclonic epilepsy i) Some of the children have absence seizures, but

this is a very different syndrome clinically ii) Myoclonic jerks on awakening iii) No 3-Hz spike-and-wave pattern

d. Prognosis 1) More than 90% of the children outgrow the seizures 2) About one-third have generalized tonic-clonic seizures

later e. Treatment

1) First-line treatment: ethosuximide, valproate, lamotrigine a) Treatment with ethosuximide only prevents

absence seizures, it does not prevent generalized tonic-clonic seizures in children with childhood absence epilepsy

b) Valproate and lamotrigine are effective for both generalized tonic-clonic seizures and absence seizures and are considered the drugs of choice in this situation

2) If monotherapy fails, combination therapy with valproate and ethosuximide

3) Anticonvulsant therapy should be stopped if the EEG is normal and the child has not had seizures for 1 to 2 years

10. Progressive myoclonic epilepsies (Table 12-6) a. Encompasses several progressive disorders, most are

lysosomal and mitochondrial disorders b. Clinical presentation

1) Progressive cognitive deterioration 2) Myoclonus (nonepileptic) 3) Seizures: tonic-clonic, tonic, or myoclonic 4) With or without ataxia, movement disorders

c. Treatment of seizures 1) First-line treatment: valproate 2) Clonazepam and lamotrigine are also used

11. Generalized epilepsy with febrile seizures plus (GEFS+) a. Clinical presentation

1) Febrile seizures 2) “Plus”: this indicates that GEFS+ occurs after age 6

years (unlike typical febrile seizures) or is associated with afebrile generalized tonic-clonic seizures (unlike typical febrile seizures)

3) One-third of patients have other seizure types: absence, myoclonic, atonic, partial seizures

b. Autosomal dominant channelopathy (Table 12-4) 1) Sodium channel (SCN) or GABAA receptor:

increased inward sodium current or decreased GABAmediated inhibition both lead to neuronal hyperexcitability a) SCN1B (chromosome 19) b) SCN1A (chromosome 2) c) SCN2A d) GABAA receptor

Ethosuximide is effective only for absence seizures

Table 12-6. Progressive Myoclonic Epilepsies

Absence seizures are driven by T-type calcium channels of the thalamus, which are blocked by ethosuximide and promoted by GABAergic drugs

Therefore, GABAergic anticonvulsants may promote absence seizures

2) Most febrile seizures, unlike GEFS+, show complex inheritance

c. EEG 1) Generalized spike-and-wave or polyspike-and-wave

pattern 12. Rasmussen’s encephalitis

a. Syndrome of chronic encephalitis with epilepsy in children

b. Intractable, progressive focal seizures; progressive hemiparesis; and cognitive deterioration

c. Radiographic characteristics of slowly progressive cortical atrophy (unilateral more common than bilateral)

d. Antibodies to GLUR3 (glutamate receptor-3) have been implicated in pathogenesis of this disorder

E. Juveniles and Adults 1. Idiopathic generalized epilepsies

a. General category in ILAE classification scheme encompassing 1) Juvenile absence epilepsy 2) Juvenile myoclonic epilepsy 3) Epilepsy with generalized tonic-clonic seizures only

2. Juvenile absence epilepsy a. Clinical presentation

1) Onset at age 10 to 17 years 2) Developmentally normal children 3) Boys and girls affected equally 4) Initially, infrequent absence seizures: usually not daily

(unlike childhood absence epilepsy) 5) Later, tonic-clonic seizures in 75% of patients upon

awakening: more common than in childhood absence epilepsy

b. Idiopathic 1) Genetic mechanism is not known 2) One-third of patients have a family history that may

include childhood absence epilepsy, juvenile absence epilepsy, juvenile myoclonic epilepsy, or epilepsy with grand mal seizures on awakening

c. EEG 1) Same as childhood absence epilepsy, except spike-and-

wave pattern may be slightly faster (3.5-4.5 Hz) and polyspikes are more frequent

d. Prognosis

1) Good: most patients are seizure-free with valproate (controls both absence and generalized seizures), ethosuximide, lamotrigine

2) Patients are less likely to outgrow seizures than those with childhood absence epilepsy a) Patients typically require lifelong anticonvulsant

therapy, and to prevent seizures as adults, patients need to avoid sleep deprivation and alcohol (as with juvenile myoclonic epilepsy)

b) Occurrence of myoclonic seizures and generalized tonic-clonic seizures make seizure remission less likely (overlaps with juvenile myoclonic epilepsy)

3. Juvenile myoclonic epilepsy a. Clinical presentation

1) Age at onset is 8 to 24 years (peak, in teens) 2) Developmentally normal 3) Boys and girls affected equally 4) Myoclonic seizures: the most frequent seizure type

a) Usually predominantly on awakening (early morning or nap)

b) Large-amplitude, bilateral, simultaneous i) Involves both arms or both legs (can cause falls) ii) Unlike nonepileptic myoclonus, it is usually

focal c) May be repetitive d) No loss of awareness e) Often, reflex seizures, including perioral reflex

myoclonia (triggered by reading, talking) 5) Most patients have less frequent generalized tonic-

clonic seizures a) Also, usually occur on awakening b) Often lead to diagnosis in patients with undiag-

nosed history of myoclonus on awakening 6) Some patients have typical absence seizures 7) Seizures are provoked by sleep deprivation, alcohol,

photic stimulation b. Idiopathic

1) One-half of patients have a family history of seizures 2) None are symptomatic (MRI is always normal) 3) No consistent genetic cause has been identified

Juvenile absence epilepsy is characterized by less frequent absence seizures than childhood absence epilepsy and more frequent (75%) development later of tonic-clonic seizures

Like juvenile myoclonic epilepsy, juvenile absence epilepsy usually requires lifelong anticonvulsant therapy and avoidance of triggers (sleep deprivation, alcohol)

Childhood absence epilepsy, in contrast, is usually outgrown

4) Overlap in the same family with other generalized epilepsy syndromes: juvenile absence epilepsy, childhood absence epilepsy, and epilepsy with grand mal seizures on awakening

c. EEG 1) Interictal

a) Three-fourths of patients have generalized 4 to 6-Hz polyspike-and-wave discharges

b) If absence seizures, EEG also has 3-Hz spike-andwave pattern

c) Often photosensitivity 2) Ictal: trains of spikes

d. Treatment 1) First-line: valproate (three-fourths of patients are

seizure-free), it controls all seizure types 2) Second-line: lamotrigine, levetiracetam, topiramate,

and zonisamide 3) Avoid carbamazepine and phenytoin, which are useful

in focal seizures but may worsen some primary generalized seizures (myoclonic and absence seizures)

e. Prognosis: it is a lifelong seizure disorder typically requiring lifelong treatment and avoidance of triggers

4. Epilepsy with grand mal seizures on awakening a. Clinical presentation

1) Onset is in second decade 2) Overlaps clinically with juvenile myoclonic epilepsy

and juvenile absence epilepsy 3) Primary seizure type is generalized tonic-clonic, usually

on awakening 4) Myoclonic or absence seizures may occur but are not

predominant b. Idiopathic/familial c. EEG, treatment, and prognosis: similar to those of

juvenile myoclonic epilepsy 5. Idiopathic photosensitive occipital lobe epilepsy

a. Clinical presentation 1) Reflex epilepsy, begins about the time of puberty 2) Occipital lobe seizures provoked by visual stimuli,

usually television or video games 3) Positive visual symptoms (occipital seizures)

a) The likely origin is the calcarine cortex b) Often, colorful rings or spots

c) May be followed by postictal negative visual symptoms d) Head turns may occur toward the visual phenomena

4) Often spreads to cause autonomic symptoms a) Epigastric discomfort, vomiting b) Can mimic migraine

5) Occasional secondarily generalized seizures b. Idiopathic c. EEG

1) Normal background 2) Interictal epileptiform activity: unilateral or bilateral

occipital ± generalized spike-and-wave, enhanced with eye closure

3) Photoparoxysmal response (epileptiform abnormalities with strobe light) is common

d. Management 1) Avoid triggers 2) First-line treatment: often valproate (as for other pri-

mary generalized epilepsies) 6. Other types of reflex seizures

a. Generalized (tonic-clonic and absence) seizures can be triggered by a flash or visual pattern 1) The most common reflex epilepsy 2) Multiple idiopathic or symptomatic generalized

epilepsy syndromes can have visually evoked seizures, these are not specific to one syndrome

b. Primary reading epilepsy: seizures occur only when patient is reading

c. Startle epilepsy 1) Seizures with unexpected sensory stimuli 2) Usually frequent seizures (tonic or clonic) 3) Often associated with developmental delay

7. Temporal lobe epilepsies a. Traditional view: temporal lobe epilepsy is usually an

acquired, symptomatic epilepsy 1) Most commonly associated with mesial temporal sclerosis 2) One-third of patients have a history of febrile seizures 3) Patients often have a familial predisposition to epilepsy,

but it is usually polygenic and/or multifactorial b. Familial forms (monogenic) of temporal lobe epilepsy

also occur 1) Onset is usually in teens or adults 2) Typically less resistant to treatment 3) A specific gene has been identified: autosomal

Focal occipital seizures of the calcarine cortex usually consist of colorful rings or spots, sometimes with head turns toward the visual phenomena

Valproate is usually effective in juvenile myoclonic epilepsy

Carbamazepine and phenytoin can worsen some types of primary generalized seizures (myoclonic, absence)

dominant partial epilepsy with auditory features (Table 12-4) a) LGI1 mutation (leucine-rich, glioma-inactivated

1 gene) in some families b) Clinical presentation: auditory auras precede com-

plex partial and secondarily generalized seizures 8. Autosomal dominant nocturnal frontal lobe epilepsy

a. Mutations in CHRNA4 gene on chromosome 20q and CHRNB2 gene on chromosome 1p, encoding proteins in M2 transmembrane segment of nicotinic acetylcholine receptors: mutations believed to cause increased acetylcholine sensitivity and channel opening (“gain-offunction” mutations)

b. Most patients have onset of seizures in first two decades (anytime between infancy to adulthood; mean age, 10 years)

c. Clusters of brief, stereotypic, nocturnal seizures, which may consist of hyperkinetic bizarre movements with clonic, tonic, or dystonic features, and sometimes, secondary generalization: clinical features similar to nonfamilial frontal lobe epilepsy

d. Seizures typically occur in non-REM sleep e. Few patients also experience stereotypic seizures during

daytime f. Not associated with cognitive deficits g. Decreased frequency and duration of seizures with time

A. Head Turn (Fig. 12-2) 1. Early nonforced head turn: ipsilateral temporal lobe

a. Voluntary-like, often with dystonic posturing of the contralateral extremity

b. May be due to neglect and/or weakness contralateral to the seizure

2. Forced head turn: contralateral hemisphere (except if it occurs after a generalized seizure) a. Prominent contraction of neck muscles, forcing the chin

to point toward the shoulder b. Tends to occur early in frontal lobe seizures, explosive

onset with other motor seizure activity c. Tends to occur later in temporal lobe seizures, as the

seizure secondarily generalizes (“late forced head turn”) d. If a forced head turn occurs after a secondarily generalized

tonic-clonic seizure, it is usually ipsilateral to seizure onset (“late ipsiversion”), probably because of the spread of seizure activity to the hemisphere contralateral to seizure onset

B. Eye Deviation 1. Accompanies forced head turns (same direction and

upward) 2. Eye deviation in isolation at seizure onset suggests

seizure activity in the occipital lobe, contralateral to the direction of gaze

C. Focal Clonic 1. Frontal and perirolandic area, contralateral hemisphere 2. Can also occur in temporal lobe complex partial

seizures-because of spread of seizure to the surrounding extratemporal cortex

D. Focal Tonic 1. Extended limb 2. Contralateral hemisphere

E. Figure 4 Sign (Fig. 12-3) 1. A tonic limb posture with one arm extended and the

Head turns may be ipsilateral to seizure onset (early nonforced head turn) or contralateral to seizure onset (forced head turn)

Autosomal dominant partial epilepsy with auditory features is an example of a monogenic temporal lobe epilepsy, caused by a mutation of the LGI1 (leucine-rich, glioma-inactivated 1) gene

other flexed at the elbow (forming a “figure 4”) 2. The seizure is lateralized to the hemisphere contralateral

to the extended arm (as with a focal tonic seizure) 3. Figure 4 sign usually occurs as the seizure generalizes 4. A forced head turn may occur (toward the extended arm)

F. Focal Dystonic 1. Sustained contorted posturing of a limb 2. Contralateral hemisphere, possibly due to spread of the

seizure to the basal ganglia

G. Fencing Posture (Fig. 12-4) 1. Seizures localize to hemisphere contralateral to the

extended arm, frontal lobe more than temporal lobe (supplementary motor area)

2. Lateral abduction and external rotation of the arm at the shoulder ± forced head turn to the side of abducted arm

3. Elbow may also flex so the hand is raised to the face

H. Ictal Paresis and Unilateral Immobile Limb 1. Seizures lateralize to contralateral hemisphere

I. Todd’s Paralysis 1. Seizures lateralize to contralateral hemisphere 2. Extratemporal lobe > temporal lobe

J. Unilateral Blinking 1. Seizures lateralize to ipsilateral hemisphere 2. Appears like winks (not forceful like a clonic facial seizure)

K. Unilateral Limb Automatism 1. Seizures lateralize to ipsilateral hemisphere 2. Simple or complex automatic behavior

a. Examples include using a pen, pulling up blanket b. The contralateral side does not take part, possibly

because of neglect

L. Postictal Nose Rubbing 1. Seizures lateralize to hemisphere ipsilateral to hand used 2. Temporal lobe more than frontal lobe

M. Postictal Cough 1. Seizures localize to temporal lobe

N. Bipedal Automatism 1. Frontal lobe more than temporal lobe

Seizures localize to the hemisphere contralateral to the extended arm in both the figure 4 sign and fencing posture

2. Bicycling or kicking

O. Hypermotor 1. Violent, restless thrashing of extremities (“hypermotor

seizures”) suggestive of frontal lobe seizures (supplementary motor area)

2. In contrast, head-and-neck thrashing suggests nonepileptic behavioral event (“pseudoseizure”)

P. Gelastic (laughter) 1. Seizures localize to the hypothalamus or mesial tempo-

ral lobe

Q. Ictal Spitting 1. Seizures localize to the right temporal lobe

R. Ictal Vomiting or Retching 1. Seizures localize to the right temporal lobe

S. Loud Vocalization 1. Frontal lobe more than temporal lobe 2. Often, nonspeech vocalization (grunting, screaming,

moaning) with frontal lobe seizures

T. Speech Arrest 1. Temporal lobe, poorly lateralizing to language-domi-

nant hemisphere 2. Speech preservation during a temporal lobe seizure sug-

gests localization in the nondominant hemisphere, but it can also occur with a dominant hemisphere seizure sparing speech areas-so speech preservation can be misleading

U. Postictal Aphasia 1. Seizures lateralize to the dominant hemisphere

V. Ictal Drooling 1. Seizures lateralize to the nondominant hemisphere

W. Postictal Confusion 1. Is usually more prominent, and persists longer after

temporal lobe seizures 2. Is not present or is brief and less prominent after

frontal lobe seizures

X. Visual Symptoms 1. Elementary: occipital lobe

a. Positive visual symptoms > negative ones but either is possible 1) Circles, spots, shapes, flashes, hemianopsia, scotoma

2) Often in color b. Pattern and progression may be stereotyped for an indi-

vidual patient c. Brief (seconds), with movement and changing shape and

size d. There may be a postictal headache, can be misdiagnosed

as migraine 1) Migraine aura has more colorful, rounded forms or

shapes rather than linear or zigzag lines 2) Migraine aura is usually briefer (seconds)

2. Complex visual hallucinations or illusions: occipitotemporoparietal junction of nondominant parietal lobe a. Relatively rare b. Are usually complex and colorful, are often previously

experienced images c. Can include micropsia (images appear smaller), meta-

morphopsia (objects appear distorted)

A. Phenytoin 1. Seizure types

a. Partial or generalized tonic-clonic seizures (primary or secondary)

b. Rarely, phenytoin can worsen other types of generalized seizures (myoclonic, absence)

2. Mechanism of action: inhibits sodium channels, especially at high rates of firing

3. Metabolism

Postictal confusion is most prominent after temporal lobe seizures and may be absent after frontal lobe seizures

Elementary visual symptoms suggest occipital lobe localization, whereas complex visual hallucinations or illusions localize to the nondominant parietal lobe or occipitotemporoparietal junction

Unilateral limb automatisms, including nose rubbing, correspond to a seizure focus ipsilateral to the limb used

a. Liver (minimal renal excretion) b. Monitor for toxicity, especially if liver disease c. May need to follow free plasma concentrations in renal

disease because of less protein binding (total may underestimate free level)

d. Nonlinear kinetics: saturable, and concentrationdependent, nonlinear (zero-order) elimination kinetics 1) As plasma concentration of the drug increases, the

elimination mechanism is progressively saturated in a nonlinear fashion, resulting in a disproportionate and logarithmic increase in plasma drug concentrations

2) Thus, small additional doses may cause a large increment in plasma concentrations when the elimination mechanism is saturated

4. Side effects a. Idiosyncratic: dyscrasias, morbilliform rash, Stevens-

Johnson syndrome, hepatitis b. Cosmetic

1) Gingival hyperplasia: lessened by good oral hygiene 2) Probably causes acne, coarse features, and hirsutism

c. Toxic to tissues (because it is highly alkaline) 1) Muscle breakdown if phenytoin is injected intramus-

cularly (therefore, only fosphenytoin is given intramuscularly)

2) Purple glove syndrome a) Severe tissue injury from constriction of blood ves-

sels, can lead to amputation or sepsis b) This occurs with intravenous administration,

occurs much less frequently with fosphenytoin d. Neurotoxicity: because of narrow therapeutic window,

blood levels are useful for monitoring for neurotoxicity (unlike newer anticonvulsants) (“narrow therapeutic window” refers to nonlinear, saturable, and concentration-dependent elimination kinetics of phenytoin, discussed above) 1) Dose-dependent, typically minimal at therapeutic levels 2) Nystagmus (at higher blood concentrations) ataxia,

dysarthria, diplopia, nausea, dizziness drowsiness, cognitive difficulties coma

3) At high levels, may rarely increase seizure frequency 4) Movement disorders

e. Vitamin-related 1) Folate deficiency/anemia 2) Increased vitamin D metabolism: causes premature

osteoporosis f. Chronic

1) Cerebellar atrophy 2) Mild peripheral neuropathy

g. Acute (with intravenous formulation) 1) Phlebitis, pain, burning sensation (with too rapid

infusion) a) This occurs because undiluted administration is

required (injectable phenytoin is insoluble in standard intravenous fluids)

b) In comparison, fosphenytoin is freely soluble in all standard intravenous fluids and thus asssociated with fewer local adverse effects such as irritation and superficial phlebitis and systemic adverse effects such as hypotension

2) Hypotension, conduction abnormalities (need to monitor blood pressure during infusion), especially if cardiac disease

3) Hypotension and conduction abnormalities are less severe with fosphenytoin

5. Drug interactions a. Variable effect in plasma concentration of warfarin b. Decreases plasma concentration of

1) Most anticonvulsants (except for gabapentin and levetiracetam): carbamazepine, oxcarbazepine, topiramate, tiagabine, lamotrigine, zonisamide, valproate, felbamate

2) Other: benzodiazepines, haloperidol, tricyclic antidepressants, oral contraceptives, digoxin, cyclosporine

c. The following increase the plasma concentration of phenytoin: cimetidine, carbamazepine, felbamate, fluconazole, fluoxetine, valproate

d. The following lower the concentration of phenytoin: valproate (decreases total, but increases free levels) and rifampin

6. Monitor: liver function tests 7. Calculating the loading dose if levels are subtherapeutic

(estimate only) a. If phenytoin level is <10 μg/mL, first-order kinetics b. Approaches zero-order kinetics above 10 μg/mL, so small

dose increments produce large increases in blood levels c. For phenytoin level <10 μg/mL, additional dose to achieve

a desired concentration can be estimated as follows: 1) Additional oral dose (mg/kg) = [volume of distribution

(0.7) desired increment (μg/mL) (desired concentration – measured concentration)]/0.92

2) Additional intravenous dose (mg/kg) = 0.7 (desired concentration – measured concentration)

3) Example: if the total phenytoin concentration is 5 μg/mL and the concentration desired is 10 μg/mL,

Phenytoin and carbamazepine act by inhibiting sodium channels, especially at high rates of firing

the additional intravenous dose will be 0.7 (10 μg/mL – 5 μg/mL) = 3.5 mg/kg

B. Fosphenytoin 1. Intravenous prodrug of phenytoin

a. Useful for intravenous load in status epilepticus or if patient is unable to take medication orally

b. Converted to phenytoin by erythrocytes (using alkaline phosphatase)

c. Fosphenytoin is a phosphate ester that is water soluble and less alkaline than phenytoin

d. In contrast, phenytoin is administered in a highly alkaline mixture of propylene glycol (antifreeze) and ethanol and can precipitate if administered too rapidly 1) Thus, fosphenytoin is less toxic to tissue, and no pur-

ple glove syndrome develops 2) Fosphenytoin can be administered more rapidly than

phenytoin and with less hypotension a) Maximal fosphenytoin dose: 150 mg of phenytoin

equivalents per minute b) Maximal phenytoin dose: 50 mg per minute

3) Because it takes 8 to 15 minutes for fosphenytoin to convert to phenytoin, the more rapid administration of fosphenytoin is offset by the time for conversion

4) Both drugs take the same time to achieve equivalent phenytoin concentration

e. Fosphenytoin also can be administered intramuscularly for a loading dose in patient who cannot take medication orally

f. Otherwise, the side effects are identical to those of phenytoin

C. Phenobarbital 1. Seizure types

a. Partial or generalized tonic-clonic seizures (primary or secondary)

b. Phenobarbital is less effective than phenytoin or carbamazepine for partial seizures

2. Mechanisms of action a. GABAA agonist b. Sodium channel antagonist c. T-type calcium channel antagonist d. Glutamate antagonist

3. Metabolism a. Liver with renal excretion of metabolites b. Increased risk of toxicity if patient has kidney or liver

disease 4. Side effects

a. Cognitive (dose-dependent) 1) Sedation, irritability, cognitive difficulties, ataxia 2) Hyperactivity (children) 3) Narrow therapeutic window, so plasma levels are use-

ful in monitoring for toxicity (unlike newer anticonvulsants)

b. Idiosyncratic: rash, aplastic anemia, agranulocytosis, thrombocytopenia, hepatitis

c. Vitamin deficiencies 1) Vitamin D deficiency and bone marrow density 2) Folate, vitamin K deficiencies

d. Cardiac and respiratory depression 5. Drug interactions

a. Decreases the plasma concentration of 1) Most anticonvulsants (except for gabapentin and leve-

tiracetam): valproate, carbamazepine, oxcarbazepine, lamotrigine, topiramate, tiagabine, zonisamide

2) Other: benzodiazepines, haloperidol, theophylline, cimetidine, warfarin, oral contraceptives

b. The following increase the plasma concentration of phenobarbital: tricyclic antidepressants and valproate

c. Cardiac and respiratory depression 6. Monitor: liver function tests

Liver enzyme-inducing anticonvulsants, like phenytoin, can reduce the effectiveness of low-dose oral contraceptives

Valproate, a liver enzyme inhibitor, also can reduce the effectiveness of low-dose oral contraceptives

Fosphenytoin is less toxic to tissue than phenytoin because fosphenytoin is water soluble

Phenytoin, in contrast, is administered in a highly alkaline mixture of propylene glycol and ethanol and can precipitate if administered too rapidly

Liver enzyme-inducing anticonvulsants include phenytoin, phenobarbital, and carbamazepine

Oxcarbazepine has less liver enzyme-induction than carbamazepine

D. Primidone 1. Seizure types

a. Partial or generalized tonic-clonic seizures (primary or secondary)

b. Primidone is less effective than phenytoin or carbamazepine for partial seizures

2. Mechanisms of action a. Multiple: sodium channel antagonist, GABAA agonist,

glutamate antagonist b. Primidone is antiepileptic, but so are its metabolites

1) Metabolized to phenobarbital and phenylethylmalonamide

2) Therefore, avoid using primidone with phenobarbital 3. Metabolism

a. Liver with renal excretion of metabolites b. Increased risk of toxicity, especially if patient has kidney

disease 4. Side effects

a. Cognitive (dose-dependent) 1) Common and prominent side effect of primidone,

may improve with time 2) Sedation, irritability, cognitive difficulties, ataxia

b. Idiosyncratic: rash, aplastic anemia, agranulocytosis, thrombocytopenia, hepatitis, lymphadenopathy

c. Vitamin deficiencies 1) Vitamin D deficiency and bone marrow density 2) Folate, vitamin K deficiencies

5. Monitor: liver function tests

E. Valproate (valproic acid and divalproex sodium) 1. Seizure types: broad spectrum

a. Partial, generalized tonic-clonic, absence, myoclonic, and tonic seizures and infantile spasms

b. Is often the drug of first choice for treating primary generalized epilepsy

2. Mechanisms of action a. Multiple: sodium channel antagonist, GABAA agonist,

T-type calcium channel antagonist b. Note: drugs with multiple mechanisms tend to have a

broad spectrum of action (compared with ethosuximide, which blocks only T-type calcium channels and has a narrow spectrum of action, i.e., only absence seizures)

3. Metabolism a. Liver (glucuronide) with renal excretion of metabolites b. Thus, valproate generally is not used if the patient has

liver dysfunction c. Can be mixed with food for near-complete absorption d. Short half-life

1) Valproic acid form must be given 3 times daily 2) Divalproex sodium can be given twice daily

4. Side effects a. Cognitive: usually minimal compared with other anti-

convulsants (sedation, irritability, cognitive difficulties, ataxia)

b. Liver 1) Encephalopathy/hyperammonemia (may occur with-

out other evidence of liver dysfunction) 2) Mildly increased transaminase levels are common: is

dose-related and reversible 3) Idiosyncratic fatal hepatitis (rare, unpredictable)

a) Most common in patients younger than 2 years, decreases with age (none in adults)

b) More common if patient receives anticonvulsant polytherapy

c) Treated with L-carnitine c. Gastrointestinal effects

1) Nausea, vomiting, gastrointestinal upset 2) Reduced if valproate is taken with food 3) Also reduced if the divalproex sodium form is used

d. Chronic effects: weight gain and thinning of hair e. Other effects

1) Menstrual irregularity, polycystic ovarian syndrome 2) Tremor (action-like essential tremor) 3) Pancreatitis: more common in children, can be fatal 4) Thrombocytopenia, platelet dysfunction: surgical

consideration 5) Teratogenic: spina bifida

f. Extensive drug interactions 1) Liver enzyme inhibitor

a) May increase the concentrations of most anticonvulsants: phenytoin, carbamazepine, phenobarbital, felbamate, lamotrigine, topiramate, tiagabine, ethosuximide

b) May decrease the concentrations of some anticonvulsants: phenytoin (total plasma concentration), oxcarbazepine

c) Unaffected: gabapentin and levetiracetam, which are also unaffected by other liver enzyme inducers or inhibitors

Valproic acid, a liver enzyme inhibitor, increases the concentrations of most anticonvulsants

Gabapentin and levetiracetam are not affected by liver enzyme inducers or inhibitors

2) Felbamate increases the valproate level 3) Liver enzyme inducers (carbamazepine, phenytoin,

phenobarbital, primidone) decrease the valproate level 4) Valproate reduces the effectiveness of low-dose oral

contraceptives 5) Interaction with phenytoin: valproate increases free

fraction of phenytoin and decreases total concentration; central nervous system levels determined by the free fraction, so the end result is increased tendency for phenytoin toxicity

5. Monitor: liver function tests

F. Carbamazepine 1. Seizure types

a. Partial or generalized tonic-clonic seizures (primary or secondary)

b. Carbamazepine can rarely worsen other types of generalized seizures (myoclonic, absence), as phenytoin does

2. Mechanism of action a. Inhibits sodium channels (like phenytoin) b. The molecule is related to that of tricyclic antidepressants

3. Metabolism a. Liver, with renal excretion of metabolites b. Use caution if patient has kidney or liver failure

4. Side effects a. Generally similar to those of phenytoin, except without

cosmetic side effects 1) Like phenytoin, carbamazepine is usually minimally

sedating 2) A loading dose cannot be given, unlike phenytoin:

the dose must be titrated up gradually to avoid toxicity 3) Like phenytoin, carbamazepine has a narrow thera-

peutic window 4) Plasma levels are useful for monitoring for toxicity

b. Autoinduction 1) The half-life decreases from 30 hours to 10 to 20

hours after the first few days to weeks of use 2) Plasma concentrations decrease in first 1 to 2 months;

it may not be necessary to increase the maintenance dose, because the epoxide metabolite also has antiepileptic action and contributes to side effects

c. Cognitive 1) Dizziness, fatigue, drowsiness, diplopia, nystagmus,

dizziness, headache, ataxia 2) Often due to starting the drug too rapidly (before

autoinduction): dose-dependent d. Idiosyncratic

1) Rash: common (10% of patients) but severe rash (Stevens-Johnson syndrome) is rare

2) Leukopenia is common: mild, clinically insignificant 3) Aplastic anemia, agranulocytosis, and thrombo-

cytopenia are rare 4) Hypersensitivity syndrome is rare: rash, eosinophilia,

lymphadenopathy, splenomegaly e. Other

1) Nausea, gastrointestinal upset (reduced if medication is taken with meals)

2) Dystonia and chorea are rare 3) Increased transaminase levels: mild, usually insignificant 4) Hyponatremia: consider if cognitive difficulties 5) Congestive heart failure 6) If taken during pregnancy, risk of spina bifida in

infant 5. Drug interactions

a. Decrease the plasma concentration of (liver enzyme induction) 1) Most anticonvulsants (except gabapentin and leve-

tiracetam): valproate, lamotrigine, tiagabine, topiramate, zonisamide, felbamate, oxcarbazepine

2) Benzodiazepines, haloperidol, theophylline, warfarin, and oral contraceptives (reduces the effectiveness of oral contraceptives)

b. Increase the plasma concentration of fluoxetine, tricyclic antidepressants, phenytoin

c. The following increase the plasma concentration of carbamazepine: erythromycin, clarithromycin, chloramphenicol, propoxyphene, verapamil

d. The following decrease the plasma concentration of carbamazepine: phenytoin, phenobarbital, primidone, and felbamate

e. Valproate increases the free fraction of both carbamazepine and its epoxide (inhibits the breakdown of the epoxide); when the level is high enough, the epoxide may also contribute to its toxicity

f. Isoniazid and carbamazepine increase the plasma concentrations of each other

6. Monitor: liver function tests, complete blood count, and sodium level

Autoinduction of carbamazepine results in reduced plasma concentrations after the first 1 to 2 months of use

The corollary is that dose-related toxicity occurs if the carbamazepine dose is escalated rapidly before autoinduction occurs

G. Oxcarbazepine 1. Chemical cousin of carbamazepine

a. Differences 1) Less liver enzyme induction

a) Thus, fewer drug interactions b) No autoinduction, thus can be titrated more rapidly

2) Metabolites have anticonvulsant effects with less toxicity a) Carbamazepine has an epoxide metabolite that is

responsible for much of its toxicity b) Oxcarbazepine has a 10-monohydroxyl metabolite

that is less toxic; anticonvulsant effect is similar to that of carbamazepine

b. Similarities: both drugs are used for the same seizure types, have the same mechanism (sodium channel antagonist), also have liver metabolism and renal excretion

2. Side effects a. Profile similar (except fewer drug interactions and less

liver enzyme induction) to that of carbamazepine, less experience

b. Interferes with contraceptives c. If patients have a carbamazepine rash, one-third will also

have an oxcarbazepine rash 3. Monitor: complete blood count, sodium levels

H. Benzodiazepines 1. Seizure types

a. Broad spectrum: partial, generalized tonic-clonic, absence, and myoclonic seizures

b. Benzodiazepines are particularly useful in status epilepticus (including absence status)

2. Mechanism of action a. Potentiates GABAA b. Enhances chloride channels, thus hyperpolarizing the

neuronal membrane and decreasing neuronal excitability 3. Metabolism

a. Liver, with renal excretion of metabolites b. Decrease dose in liver disease c. Dose is usually unchanged in kidney disease

4. Side effects a. Cognitive

1) Somnolence, drowsiness, irritability, psychosis, dysarthria, ataxia, diplopia (as with alcohol)

2) Tolerance: often used for status epilepticus or infrequent use (e.g., Diastat [rectal diazepam], is used for infrequent or severe seizures)

b. Hypoventilation: reversed with flumazenil (competitive inhibition)

c. Cardiovascular collapse d. Withdrawal syndrome (mirrors alcohol withdrawal) e. Hepatotoxicity f. Neutropenia, pancytopenia, thrombocytopenia

5. Specific benzodiazepines a. Clonazepam (long half-life): useful (but not first-line

agent) for myoclonic, absence, and partial seizures and infantile spasms

b. Lorazepam, diazepam (medium half-life): useful in status epilepticus

c. Midazolam (short half-life): anesthetic, can be used for status epilepticus

I. Ethosuximide 1. Seizure type: absence seizures only 2. Mechanism of action: antagonist of T-type calcium

channels in the thalamus 3. Metabolism: liver, with renal excretion of metabolites 4. Side effects

a. Usually well tolerated b. Cognitive

1) Drowsiness, dizziness, headache, irritability 2) Improve with tolerance

c. Gastrointestinal 1) Gastrointestinal upset, nausea, vomiting, diarrhea 2) Dose-related

d. Idiosyncratic 1) Rash, leukopenia common 2) Stevens-Johnson syndrome 3) Pancytopenia, agranulocytosis, aplastic anemia

5. Monitor: liver function tests, complete blood count

J. Felbamate 1. Seizure types: partial, generalized tonic-clonic, absence,

myoclonic, tonic, and atonic seizures 2. Metabolism: liver and kidney (half is excreted

unchanged in the urine) 3. Side effects

The primary metabolite of oxcarbazepine (10monohydroxyl metabolite) is less toxic than the primary metabolite of carbamazepine (an epoxide)

No liver autoinduction occurs with oxcarbazepine

Benzodiazepines potentiate GABAA, thus enhancing chloride channels and hyperpolarizing neurons, rendering them less excitable

a. High rates of aplastic anemia and severe, potentially fatal hepatotoxicity compared with other antiepileptic drugs 1) Required: frequent liver function tests and complete

blood count 2) Felbamate is used in only severe, refractory cases as a

last resort (often Lennox-Gastaut syndrome) b. Otherwise, mild side effects: nausea, weight loss,

insomnia 4. Drug interactions

a. Increases phenytoin, phenobarbital, and valproate concentrations and carbamazepine epoxide

b. Decreases carbamazepine level-Note: because carbamazepine epoxide (the active metabolite) increases, the carbamazepine level does not reflect activity and toxicity but underestimates them

5. Monitor: complete blood count, liver function tests, reticulocyte count

K. Tiagabine 1. Seizure types: partial or generalized tonic-clonic

seizures (primary or secondary) 2. Mechanism of action: GABA reuptake inhibitor 3. Metabolism

a. Liver b. May be used in renal failure

4. Side effects a. Cognitive: somnolence, dizziness, cognitive difficulties,

ataxia b. Gastrointestinal upset: reduced if tiagabine is taken with

meals c. Tremor d. Few drug interactions

5. Disadvantages a. Poor responder rate b. Frequent dosing (2-4 times daily)

6. No monitoring is required

L. Gabapentin 1. Seizure types

a. Partial or secondarily generalized tonic-clonic seizures b. Can worsen generalized seizures, especially myoclonic

seizures

2. Mechanism of action a. Increases postsynaptic GABA b. Dose-dependent absorption: smaller percentage of agent

is absorbed at higher doses 3. Metabolism

a. Renal excretion, essentially no metabolism before excretion

b. Longer half-life in the elderly and with renal failure 4. Side effects

a. Well tolerated b. Mild, dose-dependent toxicity c. Fatigue, somnolence, headache, ataxia, nausea d. No drug interactions e. No idiosyncratic reactions f. Excellent drug if patient is elderly or has liver failure

5. No monitoring is required

M. Lamotrigine 1. Seizure types

a. Broad spectrum: partial, generalized tonic-clonic, absence, myoclonic, tonic, and atonic seizures

b. FDA approved for Lennox-Gastaut syndrome monotherapy and for partial seizures

2. Mechanism of action a. Inhibits sodium channels, especially at high rates of

firing (like phenytoin and carbamazepine) b. Also inhibits glutamate release (unlike phenytoin and

carbamazepine, hence the broad spectrum) 3. Metabolism

a. Liver, with renal excretion of metabolites b. Liver enzyme inducers (phenytoin, carbamazepine, phe-

nobarbital) decrease the half-life of lamotrigine c. Liver enzyme inhibitor valproate increases the half-life of

lamotrigine d. Lamotrigine does not affect the metabolism of other drugs

4. Side effects a. Typically well tolerated if drug is titrated slowly

Gabapentin and levetiracetam are predominantly excreted unchanged in the urine

Both have a longer half-life in the elderly and with renal failure

Gabapentin is absorbed in a dose-dependent fashion so that a smaller proportion is absorbed at higher doses

Although felbamate is effective for a broad spectrum of seizure types, it is typically reserved for refractory epilepsy because of high rates of aplastic anemia and severe hepatotoxicity

b. Dizziness, ataxia, blurred vision, diplopia c. Rash

1) Tends to be common and more severe than with other anticonvulsants, can cause Stevens-Johnson syndrome

2) Increased risk with the following: a) Rapid titration b) Use of valproate c) Use in children

d. Can exacerbate carbamazepine toxicity (resolves with decreasing carbamazepine dose)

e. No known chronic toxicity 5. No monitoring is required

N. Levetiracetam 1. Seizure types

a. Is definitely useful for partial or generalized tonic-clonic seizures (primary or secondary)

b. May have a broad spectrum, but there is less experience with levetiracetam than with standard broad-spectrum drugs like valproate

2. Mechanism of action a. Not known precisely b. Inhibits burst firing of neurons without affecting normal

neuronal activity 3. Metabolism

a. Mostly renal (decrease dose in patients who are elderly or have renal failure)

b. Two-thirds of the drug is excreted unchanged (like gabapentin, topiramate)

c. One-third is metabolized first (acetamide hydrolysis) but no cytochrome P-450 metabolism

4. Side effects a. Well tolerated b. Rapid titration c. Cognitive

1) Usually mild 2) Drowsiness, dizziness, ataxia

d. Psychiatric

1) Only a small proportion of patients 2) Agitation, emotional lability, behavioral abnormalities

e. No important drug interactions (including oral contraceptives)

5. No monitoring is required

O. Topiramate 1. Seizure types

a. Broad spectrum b. Partial or generalized tonic-clonic seizures (primary or

secondary) c. Absence and atonic seizures: used in Lennox-Gastaut

syndrome 2. Mechanism of action

a. Multiple mechanisms like most broad-spectrum drugs b. Sodium channel inhibition, GABAA agonist, NMDA

antagonist 3. Metabolism

a. 80% is eliminated unchanged in the urine (like gabapentin, levetiracetam), some liver metabolism

b. Liver enzyme inducers increase the proportion of topiramate metabolized by the liver 1) Decreased half-life with liver enzyme inducers

(phenytoin, phenobarbital, carbamazepine) 2) Valproate has no important effect

4. Side effects a. Carbonic anhydrase inhibitor

1) Renal stone formation (as with acetazolamide and zonisamide): avoid topiramate if patient has personal or family history of stones, advise hydration, do not use with ketogenic diet

2) Paresthesias (most common adverse effect with monotherapy)

3) Angle-closure glaucoma b. Cognitive

1) Fatigue, somnolence, dizziness 2) Word-finding difficulties and mental blunting (dose-

dependent) 3) Most common adverse effects with polytherapy

c. Weight loss d. Few drug interactions e. Decreased effectiveness of oral contraceptives

5. No monitoring is required

Topiramate is a carbonic anhydrase inhibitor, like acetazolamide, and can cause renal stones, paresthesias, and angle-closure glaucoma

Lamotrigine has a broad spectrum and can be used for monotherapy in Lennox-Gastaut syndrome

Lamotrigine is associated with a particularly high risk of Stevens-Johnson syndrome, especially if used in children, titrated rapidly, and used in combination with valproate

P. Zonisamide 1. Seizure types

a. Broad spectrum b. Approved for partial seizures in adults c. Used also for generalized seizures in children

2. Mechanism of action a. Inhibits sodium channels, T-type calcium channel antagonist b. GABAA agonist

3. Metabolism a. Liver with renal excretion of metabolites b. One-third of zonisamide is excreted unchanged in the urine c. Decrease the dose if the patient is elderly or has renal failure

4. Side effects a. Carbonic anhydrase activity: renal stone formation b. Cognitive

1) Somnolence, dizziness, headache, diplopia 2) Slurred speech, mental blunting, emotional lability

c. Gastrointestinal: nausea, diarrhea, weight gain d. Rash: sulfonamide-zonisamide is contraindicated if

patient has sulfonamide allergy e. No effect on other anticonvulsant levels f. Liver enzyme inducers (carbamazepine, phenytoin,

phenobarbital) decrease the plasma concentration of zonisamide

5. No monitoring is required

Q. Vigabatrin 1. Seizure types

a. Partial and secondarily generalized seizures b. Infantile spasms

2. Mechanism of action: decreases breakdown of GABA (inhibitory neurotransmitter)

3. Not currently marketed in the United States: 30% risk of irreversible visual field constriction

R. Pregabalin 1. Approved for adjunctive therapy in partial epilepsy 2. No drug interactions 3. Low protein binding 4. Most of the drug undergoes renal clearance (as with

levetiracetam, gabapentin, and topiramate) 5. Adverse effects: drowsiness, ataxia, dizziness, weight

gain, euphoria

A. Ketogenic Diet 1. Can provide seizure relief when anticonvulsants fail

a. Typically used for children with refractory seizures,

especially myoclonic, atonic, tonic, and atypical absence seizures

b. Is effective across broad range of seizure types 2. Technique

a. Hospitalize patient for observed starvation for 4 days (until ketotic) 1) Monitor to avoid hypoglycemia 2) Goal: 4+ urine ketosis

b. The ketogenic diet is then introduced-3-4 g fat: 1 g carbohydrate + protein

3. Adverse effects a. During diet initiation: hypoglycemia, vomiting,

dehydration b. Renal stone formation: avoid carbonic anhydrase

inhibitors such as topiramate c. Vitamin, mineral, and carnitine deficiencies: supple-

ment and, before starting the diet, rule out inborn error of metabolism

d. Others: hypoproteinemia, renal tubular acidosis, hyperlipidemia, increased liver transaminase levels, pancreatitis, sepsis, long QT syndrome

B. Vagus Nerve Stimulation 1. Rationale

a. In 30% to 40% of patients, epilepsy is refractory to anticonvulsant therapy

b. Some patients are not candidates for epilepsy surgery 2. Efficacy

a. Two large randomized controlled trials showed a modest but statistically significant decrease in seizure frequency

b. The largest trial showed that high-intensity stimulation decreased seizure frequency by 28% compared with 15% for the low-intensity stimulation “active control” (presumably the placebo effect)

c. Thus, the treatment effect is only a 13% average decrease in seizure frequency

d. A smaller proportion had a 75% or greater decrease in seizure frequency

Vigabatrin is useful for controlling infantile spasms, but it is not marketed in the U.S. because of a high risk of irreversible visual field constriction

A ketogenic diet is effective for a broad range of seizure types

e. No patients were seizure-free 3. Side effects

a. Hoarseness, dyspnea, cough, and paresthesias b. Rare, device infection c. Case reports of asystole d. None of the typical central nervous system side effects of

anticonvulsants

C. Epilepsy Surgery 1. Background

a. In one-third of patients, epilepsy is refractory to anticonvulsants

b. If the epilepsy in these patients results from a focal abnormality, resective surgery may be indicated

c. Resection should be considered if adequate trials of two or three appropriate anticonvulsants prove ineffective, because additional medications are unlikely to stop the seizures

d. Early surgical consideration is recommended to avoid damaging effects of chronic seizures (central nervous system damage, psychiatric impact, socioeconomic impact)

e. Other surgical techniques can reduce seizure propagation: corpus callosotomy, multiple subpial resections

2. Surgical options (Table 12-7) a. Focal cortical resection: lesional or nonlesional b. Temporal lobectomy

1) Anterior temporal lobectomy: standard approach 2) Amygdalohippocampectomy: mesial temporal resection

a) More selective than anterior temporal lobectomy b) Useful for mesial temporal lobe epilepsy, including

mesial temporal sclerosis 3) Neocortical temporal resection

c. Multiple subpial resection 1) Used for epileptogenic focus in “critical real estate” 2) Horizontal fibers are severed to curtail spread of syn-

chronous epileptogenic activity while vertical connections are maintained so that cortical function is minimally disrupted

3) Can be used in combination with resective surgery 4) Efficacy depends on the size of the lesion, usually does

not render patients seizure-free but can reduce the intensity of the seizures

d. Hemispherectomy 1) Used if seizure activity is confined to only one poorly

functioning hemisphere (e.g., Rasmussen’s encephalopathy, Sturge-Weber disease, cerebral dysgenesis)

2) “Functional hemispherectomy” a) Central cortical resection, temporal lobectomy,

corpus callosotomy b) Disconnects the hemisphere without leaving a cavity

e. Corpus callosotomy 1) Palliative, to decrease the generalization of seizures 2) Reduces seizure intensity, frequency, and injuries

f. Deep brain stimulation 1) All forms are investigational

a) Centromedian nucleus of thalamus i) The reticulothalamocortical pathway is thought

to transmit discharges in generalized epilepsy ii) Stimulation may benefit generalized tonic-

clonic and atypical absence seizures b) Subthalamic nucleus

i) Stimulation used for Parkinson’s disease ii) In animal models and small case series in

humans, stimulation has decreased seizure activity c) Cortical stimulation: during cortical mapping, it

was observed that afterdischarges provoked by cortical stimulation can be aborted by another brief pulse of stimulation

g. Radiotherapy 1) May be antiepileptic, under investigation 2) Stereotactic radiosurgery is particularly useful for

Multiple subpial resection can be used in eloquent cortex because vertical connections subserving most cortical function are minimally disrupted

Vagus nerve stimulation typically produces a modest reduction in seizure frequency

Seizure freedom is an unrealistic goal

Table 12-7. Types of Surgical Procedures for Epilepsy

certain focal lesions (arteriovenous malformations, tumors)

3) Stereotactic radiosurgery has been used successfully for mesial temporal sclerosis, but the disadvantage is that open surgery allows intracranial monitoring to localize more accurately the patient’s seizures

3. Outcomes a. “Surgically privileged” seizure disorders

1) About 80% probability of seizure-free outcome 2) The lesion is characterized by the following

a) Well-circumscribed on MRI: this is the most critical feature

b) Well-localized interictal discharges c) Consistent focal onset by symptoms d) Focus is in noneloquent cortex: entire lesion can

be removed without major morbidity e) No other possible seizure focus or foci

b. Less ideal candidates who may still benefit 1) Nonlesional epilepsy

a) If work-up otherwise indicates the seizure is focal (e.g., video-EEG, SISCOM)

b) In nonlesional epilepsy, the outcome is better for anterior temporal lobe (60%) than for extratemporal lobe (35%-40%)

2) Bilateral mesial temporal sclerosis, if seizures originate primarily from one side

3) Multiple lesions (e.g., cavernous angiomas, tuberous sclerosis) provided that seizures are primarily from one focus (Fig. 12-5)

4) Focus near eloquent cortex: resection can be guided by cortical mapping or by operating on an awake patient

5) Large or diffuse seizure focus or lesion 6) Infantile spasms

A. Types of Status Epilepticus 1. Generalized convulsive status

a. High mortality (20%) b. Mortality varies depending on the underlying cause

2. Nonconvulsive status a. Typically evolves from prolonged seizures b. Often requires EEG to differentiate it from other causes

of unresponsiveness (postictal state, underlying neurologic disease)

3. Subtle nonconvulsive status a. Similar to nonconvulsive status b. There is subtle face, eyelid, or eye twitching

4. Focal status (simple or complex) a. Complex focal status has significant morbidity and

mortality b. It merits aggressive treatment

5. Absence status a. Benign b. Brain damage is unlikely

Stereotactic radiosurgery has been used successfully for mesial temporal sclerosis, but the disadvantage is that intracranial monitoring cannot be performed

Patients whose seizures are associated with a single, wellcircumscribed MRI lesion, have well-localized interictal discharges, produce consistent symptoms, and are located in noneloquent cortex have a high probability of freedom from seizures (~80%) after resection of the seizure focus

c. Responds to low-dose benzodiazepines 6. Myoclonic status

a. Usually indicates severe neurologic injury (e.g., anoxic, degenerative)

b. Thus, treatment response and prognosis are poor

B. Generalized Convulsive Status Epilepticus 1. Definition

a. Classic definition: 30 minutes of continuous seizure activity or two or more seizures in 30 minutes without recovery of consciousness

b. Controversial new definition: more than 5 minutes of seizure activity or two or more seizures without recovery of consciousness

c. Advocates for old definition point out that neuronal damage begins after 30 minutes

d. Advocates for new definition point out that seizures usually stop in less than 2 minutes

e. All agree that early treatment is essential because longer seizures are more difficult to stop and can cause neuronal injury within 20 to 30 minutes, leading to mesial temporal sclerosis and epilepsy

2. Management (for algorithmic approach to treatment of status epilepticus, see Chapter 10) a. Initial management (adult)

1) Airway, breathing, circulation a) Ventilation is usually adequate if an airway is main-

tained (e.g., with oral airway) b) Electrocardiographic and blood pressure monitor-

ing: potential cardiac side effects of anticonvulsants (hypotension, arrhythmias)

2) Oxygen, pulse oximetry 3) If the intravenous route is not possible, consider the

rectal or intramuscular route 4) Intubation: usually required only after administration

of large doses of anticonvulsants that have sedative properties a) If the patient is intubated, use short-acting nonde-

polarizing neuromuscular blockade (e.g., rocuronium) b) Avoid succinylcholine because it causes hyper-

kalemia and arrhythmias, especially if the patient has rhabdomyolysis from the convulsions

5) Check blood glucose: if level is low, give 50 mL of

50% glucose with 100 mg thiamine b. Pharmacologic management: overview

1) First-line agent: benzodiazepines 2) Second-line agent: phenytoin 3) Third-line agent: phenobarbital vs. anesthetic agent

(midazolam, propofol) 4) If refractory to midazolam or propofol, consider

inhalent anesthetic (isoflurane is the first choice) 3. Medications for convulsive status epilepticus

a. See above for further details about these drugs, this section reviews issues particularly relevant to their use in status epilepticus

b. Lorazepam 1) Probably the preferred benzodiazepine because it not

only has an onset of action as fast as that of diazepam but also a longer duration of action (24 hours vs. 30 minutes)

2) Dose (adults): 0.1 mg/kg intravenously at 1 mg/min 3) Adverse effects: respiratory depression, hypotension,

decreased level of consciousness 4) Also rectal, sublingual, and intramuscular preparations 5) Can also be given as a continuous intravenous infu-

sion for refractory status epilepticus, like midazolam but with less tachyphylaxis, but long offset (especially with prolonged use)

c. Diazepam 1) Alternative to lorazepam but risk of recurrent seizure

because of short duration of action 2) Dose (adults): 0.2 mg/kg intravenously at 2 mg/min 3) Rectal gel useful in children and infants or if there is

no intravenous access a) 2 to 5 years old: 0.5 mg/kg b) 6 to 11 years old: 0.3 mg/kg c) 12 years and older: 0.2 mg/kg (adult intravenous

dose) 4) Adverse effects: respiratory depression, hypotension,

decreased level of consciousness d. Phenytoin

1) Standard second-line agent, but consider valproate if there are cardiac issues

2) Water insoluble, so formulation is alkaline (pH 12) with propylene glycol and ethanol (responsible for

Depolarizing neuromuscular blocking agents (e.g., succinylcholine) are contraindicated in patients with generalized convulsive status epilepticus because of the risk of hyperkalemia and arrhythmias

Myoclonic status epilepticus typically reflects severe neurologic injury (e.g., anoxic or degenerative)

Treatment response and prognosis are poor

tissue toxicity and purple glove syndrome and, to some degree, cardiac effects)

3) Dose (adults) a) 20 mg/kg intravenously at 50 mg/min b) May repeat an additional 10 mg/kg if seizures

continue 4) Note that administration is slow because of the risk of

cardiovascular side effects (hypotension, arrhythmias, QT prolongation) a) For a 70-kg man receiving 1.4 g, administration

would take 28 minutes b) Electrocardiographic and blood pressure monitor-

ing are essential (intensive care unit or emergency department) i) Slow rate of administration if hypotension or

cardiac effects occur ii) In part, cardiac effects are due to the propylene

glycol diluent, so fewer cardiac effects occur with fosphenytoin

5) Adverse effects: hypotension, QT prolongation, purple glove syndrome, others (see above)

e. Fosphenytoin 1) More expensive than phenytoin, but better side-effect

profile 2) It is a phosphate ester prodrug of phenytoin

a) It is water soluble, with no risk of purple glove syndrome and less risk of cardiac adverse effects

3) Dose: 20 mg/kg phenytoin equivalents intravenously at 150 mg/min a) Despite faster infusion rate, time to onset of action

is similar to that for phenytoin because of conversion time

b) Cardiac monitoring is still needed, slow the rate of administration if hypotension or arrhythmia occurs

f. Valproate 1) Alternative, second choice to phenytoin 2) Loading dose: 15 to 20 mg/kg (over 5 minutes) 3) Efficacy has been poorly studied, but side-effect

profile is favorable compared with phenytoin if patient has hemodynamic instability

g. Phenobarbital 1) Dose: 20 mg/kg intravenously at 50 mg/min (same as

for phenytoin) 2) Marked side effects

a) Use of phenobarbital after phenytoin fails to control seizures is controversial because of its adverse sideeffect profile i) Depresses respiratory drive and consciousness ii) High risk of hypotension, decreased cardiac

contractility iii) Because of the long half-life (48 hours), the

effects are prolonged h. Midazolam

1) A benzodiazepine with a short half-life, used as an intravenous drip for its anesthetic properties in refractory status epilepticus

2) Dose: 0.2 mg/kg load over 1 minute (intubate), then 0.05 to 2.0 mg/kg per hour

3) Advantages: fast onset and offset, well tolerated, less hypotension than with phenobarbital or propofol

4) Disadvantages: rapid onset of tachyphylaxis i. Propofol

1) Anesthetic: GABAA agonist (like benzodiazepines) 2) Dose: 3 to 5 mg/kg load (intubate), then 1 to 15

mg/kg per hour 3) Advantages: short onset and offset, effective 4) Disadvantages: bradycardia, hypotension, high lipid

content, slower offset after prolonged infusion, propofol infusion syndrome

5) Propofol infusion syndrome a) Triad: severe hypotension, lipidemia, and meta-

bolic acidosis b) Most common in children with metabolic enzyme

deficiencies, but it can occur in adults

Lorazepam has an onset of action as fast as that of diazepam but has a longer duration of action

Although fosphenytoin can be administered more rapidly than phenytoin, the time to onset of action is similar because of fosphenytoin’s conversion time

Propofol infusion syndrome, most commonly seen in children with metabolic enzyme deficiencies, is the triad of hypotension, hyperlipidemia, and metabolic acidosis

The use of phenobarbital as a third-line agent in status epilepticus (before anesthetics such as midazolam and propofol) is controversial because of the long half-life and the risks of hypotension and decreased cardiac contractility

A. Hormonal Abnormalities Related to Seizures and Anticonvulsants (infertility, menstrual irregularities, ovarian dysfunction, cosmetics)

1. Generalized or complex partial seizures can propagate to the hypothalamus, altering hormone release

2. Anticonvulsants can influence metabolism of sex hormones a. This is often mediated by cytochrome P-450 or by

increased protein binding b. They can directly influence the cortical input to the

hypothalamic-pituitary-ovarian axis c. Liver enzyme inducers (e.g., carbamazepine, phenytoin,

phenobarbital, topiramate) reduce steroid hormone levels 3. Result of seizures or drug effects

a. Anovulatory cycles, infertility, menstrual irregularities b. Polycystic ovary-like syndrome

4. Increased androgen levels can produce hirsutism, acne, excess facial hair, and weight gain a. Some debate about what hormonal dysfunction is attrib-

utable to drugs vs. epilepsy b. Polycystic ovary-like syndrome has been attributed to

valproate, but some argue that the seizures are the culprit

B. Hormonal Exacerbation of Seizures 1. Estrogen has proconvulsant effects

a. Estrogen downregulates GABAA and GABAA receptor synthesis, thus increasing neuronal excitability by decreasing GABA-mediated inhibition

b. Estrogen also is an NMDA agonist in the hippocampus (excitatory)

2. Progesterone has anticonvulsant effects (opposite effects to estrogen) a. Upregulates GABAA and GABAA receptor synthesis b. Antagonist of hippocampus NMDA receptors

3. With increased estrogen, seizures may be triggered during menarche

4. Catamenial seizures: occur primarily during perimenstrual period when estrogen levels are high relative to progesterone levels

5. Progesterone therapy can be used in some circumstances of increased estrogen:progesterone ratio for antiepileptic properties a. Example: with anovulatory cycle, progesterone produc-

tion is inadequate because the corpus luteum (which produces progesterone) does not form from the follicle (which produces estrogen), so if a patient has seizures associated with anovulatory cycles, progesterone therapy can be helpful

C. Efficacy of Oral Contraceptives 1. Anticonvulsants that increase the metabolism of oral

contraceptives reduce the efficacy of these medicines: carbamazepine, phenytoin, phenobarbital, oxcarbazepine, and topiramate (at high doses) a. Thus, the typical low-dose pills have a higher failure rate b. Higher dose pills or additional contraceptive methods

are recommended 1) Example: carbamazepine reduces estradiol by 40% to

50% 2) Thus, an oral contraceptive pill containing 50 μg

ethinyl estradiol is recommended instead of the typical “low-dose” 35 μg

2. The medicines that have no interaction with oral contraceptives are valproate, gabapentin, pregabalin, tiagabine, levetiracetam, zonisamide

D. Pregnancy: maternal issues 1. 25% of women have worsening of seizure frequency

during pregnancy a. Increased risk if seizure frequency was high before pregnancy b. Pharmacokinetics change during pregnancy (e.g.,

increase in metabolism volume of distribution and protein binding), decreasing free drug levels 1) Follow free drug levels during pregnancy 2) Lamotrigine levels decrease markedly during pregnan-

cy compared with those of other anticonvulsants c. The risk of eclampsia, preeclampsia, and other maternal

complications of pregnancy is debated but may be increased

E. Pregnancy: fetal issues 1. Risks to the fetus

a. Most (>90%) babies born to epileptic women are healthy b. Risks: miscarriage, prematurity, cerebral palsy, develop-

mental delay, epilepsy, and major malformations (cardiac and neural tube defects)

Free anticonvulsant levels generally decrease during pregnancy because of increase in metabolism, volume of distribution, and protein binding

Estrogen has proconvulsant effects and progesterone has anticonvulsant effects

c. Previously referred to as “fetal hydantoin syndrome,” now called “fetal anticonvulsant syndrome”

d. Risk of major fetal malformation is 4% to 8% (double the risk for the general population) 1) Various malformations are seen with most anti-

convulsants a) Valproate and carbamazepine are associated partic-

ularly with spina bifida (1%-2% with valproate use, 0.5% with carbamazepine)

2) Greatest risk of cleft lip and palate, neural tube malformations, and congenital heart defects is during the first trimester

3) Note that the neural tube closes during weeks 3 to 4-a critical period

4) Folate supplementation decreases birth defects (especially neural tube defects) in the general population a) Whether folate also protects against defects related

to anticonvulsants is not known b) The Centers for Disease Control and Prevention

recommends 0.4 mg/day for all women planning pregnancy

c) Higher doses are often recommended for women taking anticonvulsants (up to 5 mg/day, but the optimal dose is not known)

d) Folate should be started while the woman is attempting pregnancy because the neural tube closes before many women know they are pregnant

e. Although anticonvulsants are associated with increased risks of congenital malformations, recurrent seizures are considered by most neurologists as unacceptable risks to the fetus and mother

f. Seizures can cause fetal and placental trauma and hypoxia g. Risks of anticonvulsant-induced congenital malforma-

tions increase with the number of drugs used (5% for 1 drug, 23% for 4 drugs)

h. Goal: seizure control with the fewest anticonvulsants

necessary and at the lowest dose i. There may also be an increased risk of fetal hemorrhage

from internal bleeding, thought possibly due to vitamin K-dependent clotting factor deficiency from anticonvulsants 1) Babies are routinely given vitamin K intramuscularly 2) Consider additional oral vitamin K during the last

month of pregnancy 2. Breast-feeding

a. Highly protein-bound anticonvulsants are expressed to a greater degree in breast milk 1) These can cause sedation, especially barbiturates 2) Withdrawal symptoms can occur when nursing is

discontinued b. Anticonvulsants rarely cause hematologic and liver side

effects c. For newer anticonvulsants, the risks of nursing are uncertain d. Benefits of nursing may outweigh potential neurologic

consequences

F. Osteoporosis 1. Liver enzyme-inducing anticonvulsants decrease the

level of active vitamin D, leading to premature osteoporosis and bone fractures

2. Epileptics are at risk for poor bone health independent of the effects of liver enzyme-inducing drugs

3. Bone densitometry is recommended for patients receiving long-term anticonvulsant therapy

4. Consider weight-bearing exercises, vitamin D and calcium supplementation, and bisphosphonate therapy

A. Structural Imaging 1. MRI is more sensitive than CT (especially with 15-mm

coronal cuts through temporal lobes) a. MRI is useful for evaluating the cause of new-onset

seizure disorder b. Common lesional etiologies include stroke, traumatic

injury, tumor, malformation of cortical development, cysticercosis, mesial temporal sclerosis (Fig. 12-6)

2. CT is useful in emergent settings to rule out acute blood 3. When structural imaging is indicated:

a. First-time seizure if focal lesion is suspected 1) Especially consider structural imaging if patient is

older than 40 years (idiopathic epilepsy is unlikely in this age group)

2) Especially consider structural imaging if patient had a

The goal in pregnancy is to control seizures with the fewest necessary antiepileptic drugs at the lowest doses needed

Uncontrolled seizures are risky to the fetus

Folate use is recommended for epileptic women before conception and during the first trimester because the neural tube closes between weeks 3 and 4

focal seizure (suggests focal lesion) b. Children with first nonfebrile seizure

1) Urgent neuroimaging if there is focal deficit or prolonged confusion

2) Nonurgent MRI if there is cognitive or motor impairment of unknown cause, abnormal neurologic findings, seizure of focal onset, EEG is not consistent with an idiopathic epilepsy syndrome, or patient is younger than 1 year

c. ILAE advocates imaging for all epilepsy patients at some point unless they clearly have an idiopathic epilepsy syndrome (consistent clinical story and EEG with normal examination findings)

B. Functional Imaging 1. Indicated in epilepsy surgical evaluation when primary

means of seizure focus localization are inconclusive (history, examination, interictal EEG, MRI, video-EEG)

2. Is often used to provide a target for intracranial electrode placement

3. Positron emission tomography (PET) (Fig. 12-7) a. Images the degree of uptake of various radioactive

ligands

b. The usual glucose-uptake ligand is [18F]fluoro-2-deoxyD-glucose 1) Reflects metabolic activity 2) Its short half-life limits use ictally 3) Usually interictal, to search for hypometabolic regions

that are potentially epileptogenic regions 4) A seizure occurring during PET may show a hyper-

metabolic region c. Advantages

1) If abnormal, high concordance with EEG 2) Particularly useful in infants with infantile spasms

because hypometabolic regions of these patients can be resected, with excellent surgical outcome despite normal structural imaging (80% in one study)

d. Disadvantages 1) Less likely to be abnormal if MRI is normal 2) Interictal, so does not reflect seizure activity 3) Expensive, need a cyclotron to generate radiotracer 4) Abnormalities are often diffuse, e.g., abnormality

often lateralizes to one hemisphere but not within the hemisphere

4. Single photon emission computed tomography (SPECT) and subtraction ictal SPECT coregistered to

PET scans show interictal hypometabolism, whereas ictal SPECT scans show ictal increases in blood flow

MRI (SISCOM) (Fig. 12-7 and 12-8) a. Ictal SPECT measures increased blood flow during a

seizure b. Unlike SISCOM, ictal SPECT is used at many institutions c. Interictal SPECT measures decreased blood flow in an

epileptogenic region d. Interictal SPECT is less sensitive than ictal SPECT e. Comparison of ictal and interictal SPECT facilitates

identification of ictal abnormalities by contrast f. SISCOM takes the ictal image and subtracts it digitally

from the interictal image 1) This “difference” image is coregistered to MRI to

anatomically localize the seizure focus 2) SISCOM localization is an independent predictor of

epilepsy surgery outcome

3) If EEG and MRI are nonlocalizing, the surgical outcome is better if the SISCOM abnormality is resected (60% vs. 20%)

g. Advantages 1) Less expensive than PET, no cyclotron is needed 2) Radiotracers have longer half-life, so ictal injections

are feasible

SISCOM can often localize a seizure focus even when MRI and EEG are nonlocalizing

However, only a seizure focus active within 1 minute after injection is reflected

h. Disadvantages 1) Requires prompt injection during seizure, within

1 minute 2) If the injection is delayed, then postictal hypoperfu-

sion is imaged, which has a wider distribution 3) Reflects only the seizure injected, not necessarily

patient’s every seizure type 5. Magnetic resonance spectroscopy (MRS)

a. Noninvasive measurement of chemical composition of brain

b. MRS measures hydrogen nucleus, thereby measuring metabolites such as N-acetylaspartate (NAA), creatine, and choline 1) NAA reflects neuron abundance 2) Creatine and choline are associated with glial cells

3) NAA:creatine ratio a) If decreased in mesial temporal lobe, suggests

mesial temporal sclerosis b) Especially helpful in temporal lobe resection candi-

dates who do not have hippocampal atrophy c) Seizures usually arise from the side with relatively

decreased NAA, reflecting mild mesial temporal sclerosis

d) If MRS shows bilateral hippocampal abnormalities, seizure outcome of temporal lobectomy is less favorable and postoperative memory problems are more substantial

6. Functional cortex mapping a. Intracarotid injection of amobarbital (Wada’s test)

1) Depresses cortical function of hemisphere ipsilateral

3. Delayed development 4. Day-care attendance 5. Incidence does not increase in proportion to increase in

temperature 6. No identifiable risk factor in approximately 50% of patients

G. Risk Factors For Developing Afebrile Epilepsy After Febrile Seizure

1. Developmental delay and abnormal neurologic examination

2. Family history of afebrile seizures 3. Complex febrile seizure

H. Risk of Recurrent Febrile Seizures Is Increased in Presence of

1. Family history of febrile or unprovoked seizures 2. Low temperature at time of initial febrile seizure 3. Initial febrile seizure when child is younger than 12

months (50% of patients younger than 1 year presenting with a first febrile seizure will have recurrence, as compared with 20% of children older than 3 years at time of first febrile seizure)

4. Not increased with complex febrile seizure on presentation as compared with simple febrile seizure

I. Risk of Epilepsy 1. <5% of patients with febrile seizures develop epilepsy 2. About 15% of patients with epilepsy have a history of

febrile seizures

J. Simple Febrile Seizure 1. Most febrile seizures 2. Brief (<15 minutes) 3. Generalized, lack focality 4. Occur in neurologically normal patients 5. Not associated with persistent deficits 6. Often have a familial predisposition: may be autosomal

dominant with variable penetrance, polygenic, or autosomal recessive

K. Complex Febrile Seizure 1. 20% of febrile seizures 2. Prolonged (>15 minutes) 3. May have focal features 4. Seizure recurrence less than 24 hours 5. Abnormal neurologic examination 6. Postictal neurologic signs (e.g., Todd’s paralysis) 7. Complex febrile seizures are more likely to be due to

meningitis, encephalitis, or underlying seizure disorder than simple complex seizures

to carotid artery injected 2) Useful in identifying hemisphere of language

localization, but poor identification of specific anatomical structures involved

b. Functional MRI 1) Can be used to measure task-activated cortex, thereby

localizing function more accurately than with the amobarbital test

2) It measures changes in deoxyhemoglobin on T2weighted MRI

3) A decrease of deoxyhemoglobin reflects an increase in blood flow in activated brain

4) Functional MRI may be advantageous compared with electrocortical stimulation because it is noninvasive

A. Prevalence: 3% to 5% of all children, usually benign outcome

B. Age 1. Usually between 6 months and 5 years (range, 1 mo-10

years) 2. 90% occur within the first 3 years of life

C. Typically Associated With Common Childhood Infections (including human herpesvirus-6)

D. Any Clinical Features Indicative of Central Nervous System Infection Preclude the Diagnosis

E. Recurrence 1. About 1/3 of patients with one febrile seizure have at

least one additional seizure (higher risk if child is younger than 12 months at time of first febrile seizure)

2. Half of this group (15% of all patients) have a second seizure, and 9% of all patients have 3 or more recurrent seizures

F. Risk Factors for Developing Febrile Seizure 1. Family history

a. Febrile seizures are 2 or 3 times more likely to occur in family members of affected children

b. Increased risk with family history of febrile seizures (firstdegree relatives), higher risk with family history in both parents

c. Increased risk of afebrile epilepsy in first-degree relatives of patients with febrile seizures

2. Prolonged hospitalization in neonatal intensive care unit

L. Prophylaxis After Febrile Seizure 1. Usually not indicated, but may be considered for recurrent

or prolonged febrile seizures, later afebrile epilepsy, or after a complex partial seizure, especially in patients with abnormal neurologic examination and developmental delay

2. Chronic antiepileptic drug prophylaxis: phenobarbital and valproate a. Recurrence reduction; however, prophylaxis is usually

avoided because drug side effects are a concern b. Phenobarbital can cause irritability, hyperactivity, somno-

lence, and possible interference with cognitive development

3. Short-term antiepileptic drug prophylaxis (prophylaxis during febrile illnesses only) a. Diazepam

1) Studied in children (mean age, 2 years) with a history of febrile seizures

2) Significant seizure reduction 3) Significant side effects: irritability, lethargy, ataxia 4) Useful if frequent or prolonged febrile seizures

b. Ibuprofen: randomized placebo-controlled studies of ibuprofen use during febrile illnesses have failed to show statistically significant benefit

1. In benign myoclonus of infancy, the EEG is characterized by: a. 3-Hz spike-and-wave pattern b.Atypical spike-and-wave pattern c. Normal pattern d.Polyspike-and-wave pattern e. Hypsarrhythmia

2. Mutations of KCNQ2 on chromosome 20 are seen in: a. Early myoclonic encephalopathy b.Benign familial neonatal convulsions c. Aicardi’s syndrome d.Benign childhood epilepsy with centrotemporal spikes e. Juvenile myoclonic epilepsy

3. Medications most likely to worsen absence seizures are: a. GABAergic agent b.Sodium channel inhibitor c. T-type calcium channel antagonist d.Glutamate antagonist e. Anesthetic agent

4. Automatisms can be seen in: a. Partial complex seizures b.Absence seizures c. Atypical absence seizures d.All the above e. a and b

5. A seizure beginning gradually with an early, nonforced left head turn, later followed by a forced right head turn, most likely localizes to the: a. Left frontal lobe b.Left temporal lobe c. Right frontal lobe d.Right temporal lobe e. a or b

6. Which of the following is suggestive of a nonepileptic behavioral event rather than a seizure? a. Thrashing of extremities b.Thrashing of head and neck c. Ictal laughter d.Ictal grunting or moaning e. Ictal spitting

7. If a patient’s blood level of phenytoin is 2 mg/mL and

the desired blood level is 12 μg/mL, what additional intravenous dose should be given? a. 20 mg/kg b.14 mg/kg c. 8 mg/kg d.6 mg/kg e. 4 mg/kg

8. Which of the following can reduce the effectiveness of low-dose oral contraceptives? a. Phenytoin b.Valproate c. Oxcarbazepine d.All the above e. None of the above

9. Drug concentrations of which of the following anticonvulsants are not affected by liver enzyme-inducing and inhibiting drugs? a. Lamotrigine and topiramate b.Oxcarbazepine and gabapentin c. Levetiracetam and gabapentin d.All the above e. None of the above

10.Felbamate is particularly associated with a high rate of complications due to: a. Liver autoinduction b.Stevens-Johnson syndrome c. Purple glove syndrome d.Worsening of absence seizures e. Aplastic anemia and severe hepatotoxicity

11.Hypotension, lipidemia, and metabolic acidosis can be seen with infusion of: a. Propofol b.Phenobarbital c. Lorazepam d.Fosphenytoin e. Midazolam

12.When advising an epileptic woman about pregnancy, which of the following would not be true? a. The benefits of nursing probably outweigh the risks

for some anticonvulsants b.Most (>90%) babies born to epileptic women are

healthy c. Free drug levels often decrease during pregnancy

d.The incidence of spina bifida is increased with valproate and carbamazepine therapy

e. Folate supplementation has been proved to prevent spina bifida caused by valproate

13.A SISCOM (subtraction ictal SPECT coregistered to MRI) focus reflects: a. A hypermetabolic focus b.A hyperperfused focus c. A relative ictal focus of hyperperfusion, as compared

with interictal SPECT d.A relative ictal hypermetabolic focus, as compared

with interictal SPECT e. None of the above

14.All the following are positive predictors of freedom from seizures following epilepsy surgery except: a. Circumscribed MRI lesion b.Focus in eloquent cortex c. Focal interictal discharges d.Consistent semiology e. Focal SISCOM abnormality

15.Antibodies to which of the following have been implicated in the pathogenesis of Rasmussen’s encephalitis? a. GABA receptors b.Nicotinic acetylcholine receptors c. Voltage-gated sodium channels d.Glutamate receptors

1. Answer: c. Benign myoclonus of infancy is a nonepileptic disorder with a normal EEG. The symptoms can mimic infantile spasms. However, the characteristic interictal hypsarrhythmia pattern of infantile spasms is not present in benign myoclonus of infancy.